Slot System
Featured Buckets
Featured Buckets Admin

When Can Exercise Supplant Surgery for Degenerative Meniscal Tears?

Article Type
Changed
Fri, 05/05/2017 - 01:24
Display Headline
When Can Exercise Supplant Surgery for Degenerative Meniscal Tears?

 

A 48-year-old man presents to your office for follow-up of right knee pain that has been bothering him for the past 12 months. He denies any trauma or inciting incident for the pain. On physical exam, he does not have crepitus but does have medial joint line tenderness of his right knee. An MRI shows a partial medial meniscal tear. Do you refer him to physical therapy (PT) or to orthopedics for arthroscopy and repair?

The meniscus—cartilage in the knee joint that provides support, stability, and lubrication to the joint during activity—can tear during a traumatic event or as a result of degeneration over time. Traumatic meniscal tears typically occur in those younger than 30 during sports (eg, basketball, soccer), whereas degenerative meniscal tears generally occur in patients ages 40 to 60.2,3 The annual incidence of all meniscal tears is 79 per 100,000.4 While some clinicians can diagnose traumatic meniscal tears based on history and physical examination, degenerative meniscal tears are more challenging and typically warrant an MRI for confirmation.3

Meniscal tears can be treated either conservatively, with supportive care and exercise, or surgically. Unfortunately, there are no national orthopedic guidelines available to help direct care. In one observational study, 95 of 117 patients (81.2%) were generally satisfied with surgical treatment at four-year follow-up; satisfaction was higher among those with a traumatic meniscal tear than in those with a degenerative tear.5

Two systematic reviews of surgery versus nonoperative management or sham therapies found no additional benefit of surgery for meniscal tears in a variety of patients with and without osteoarthritis.6,7 However, both studies were of only moderate quality, because of the number of patients in the nonoperative groups who ultimately underwent surgery. Neither of the studies directly compared surgery to nonoperative management.6,7Another investigation—a multicenter, randomized, double-blind, sham-controlled study conducted in Finland involving 146 patients—compared sham surgery to arthroscopic partial meniscectomy. Both groups received instruction on performing post-procedure exercises, and both groups had similar and marked improvement in pain and function.8

Clinical practice recommendations devised from a vast systematic review of the literature recommend that the decision for surgery be based on patient-specific factors, such as symptoms, age, mechanism of tear, extent of damage, and occupational/social/activity needs.9

STUDY SUMMARY

Exercise is as good as surgery

The current superiority RCT compared exercise therapy to arthroscopic partial meniscectomy. Subjects (ages 35 to 60) presented to the orthopedic department of two hospitals in Norway with unilateral knee pain of more than two months’ duration and an MRI-delineated medial meniscal tear. They were included in the study only if they had radiographic evidence of minimal osteoarthritis (Kellgren-Lawrence classification grade ≤ 2). Exclusion criteria included acute trauma, locked knee, ligament injury, and knee surgery in the same knee within the previous two years.

The primary outcomes were change in patient-reported knee function (as determined by overall Knee injury and Osteoarthritis Outcome Score [KOOS] after two years) and thigh muscle strength at three months (as measured by physiotherapists). The researchers used four of the five KOOS subscales for this analysis: pain, other symptoms (swelling, grinding/noise from the joint, ability to straighten and bend), function in sports/recreation, and knee-related quality of life (QOL). The average score of each subscale was used.

Secondary outcomes included the five individual KOOS subscales (the four previously mentioned, plus activities of daily living [ADLs]), as well as thigh muscle strength and lower-extremity performance test results.

Methods. Testing personnel were blinded to group allocation; participants wore pants or neoprene sleeves to cover surgical scars. A total of 140 patients were randomized to either 12 weeks (24-36 sessions) of exercise therapy alone or a standardized arthroscopic partial meniscectomy; upon discharge, those in the latter group received written and oral encouragement to perform simple exercises at home, two to four times daily, to regain range of motion and reduce swelling.

Results. At two years, the overall mean improvement in KOOS4 score from baseline was similar between the exercise group and the meniscectomy group (25.3 pts vs 24.4 pts, respectively; mean difference [MD], 0.9). Additionally, muscle strength (measured as peak torque flexion and extension and total work flexion and extension) at both three and 12 months showed significant objective improvements favoring exercise therapy.

In the secondary analysis of the KOOS subscale scores, change from baseline was nonsignificant for four of the five (pain, ADL, sports/recreation, and QOL). Only the symptoms subscale had a significant difference favoring exercise therapy (MD, 5.3 pts); this was likely clinically insignificant on a grading scale of 0 to 100.

Of the patients allocated to exercise therapy alone, 19% crossed over and underwent surgery during the two-year study period.

 

 

 

WHAT’S NEW

Head-to-head comparison adds evidence

This is the first trial to directly compare exercise therapy to surgery in patients with meniscal tears. Interestingly, exercise therapy was as effective after a two-year follow-up period and was superior in the short term for thigh muscle strength.1

The results of this study build on those from the aforementioned smaller study conducted in Finland.8 In that study, both groups received instruction for the same graduated exercise plan. The researchers found that exercise was comparable to surgery for meniscal tears in patients with no osteoarthritis.

CAVEATS

What about more severe osteoarthritis?

This trial included patients with no to mild osteoarthritis in addition to their meniscal tear.1 It is unclear if the results would be maintained in those with more advanced disease. Additionally, 19% of patients crossed over from the exercise group to the surgery group, even though muscle strength improved. Therefore, education about the risks of surgery and the potential lack of benefit is important.

CHALLENGES TO IMPLEMENTATION

Cost and effort of PT

The cost of PT can be a barrier for patients who have adequate insurance coverage for surgery but inadequate coverage for PT. Additionally, exercise therapy requires significant and ongoing time and effort, which may deter those with busy lifestyles. Patients and clinicians may view surgery as an “easier” fix.

ACKNOWLEDGEMENT

The PURLs Surveillance System was supported in part by Grant Number UL1RR024999 from the National Center For Research Resources, a Clinical Translational Science Award to the University of Chicago. The content is solely the responsibility of the authors and does not necessarily represent the official views of the National Center For Research Resources or the National Institutes of Health.

Copyright © 2017. The Family Physicians Inquiries Network. All rights reserved.

Reprinted with permission from the Family Physicians Inquiries Network and The Journal of Family Practice (2017;66[4]:250-252).

References

1. Kise NJ, Risberg MA, Stensrud S, et al. Exercise therapy versus arthroscopic partial meniscectomy for degenerative meniscal tear in middle aged patients: randomised controlled trial with two year follow-up. BMJ. 2016;354:i3740.
2. Beals CT, Magnussen RA, Graham WC, et al. The prevalence of meniscal pathology in asymptomatic athletes. Sports Med. 2016;46:1517-1524.
3. Maffulli N, Longo UG, Campi S, et al. Meniscal tears. Open Access J Sports Med. 2010;1:45-54.
4. Peat G, Bergknut C, Frobell R, et al. Population-wide incidence estimates for soft tissue knee injuries presenting to healthcare in southern Sweden: data from the Skåne Healthcare Register. Arthritis Res Ther. 2014;16:R162.
5. Ghislain NA, Wei JN, Li YG. Study of the clinical outcome between traumatic and degenerative (non-traumatic) meniscal tears after arthroscopic surgery: a 4-years follow-up study. J Clin Diagn Res. 2016;10:RC01-RC04.
6. Khan M, Evaniew N, Bedi A, et al. Arthroscopic surgery for degenerative tears of the meniscus: a systematic review and meta-analysis. CMAJ. 2014;186:1057-1064.
7. Monk P, Garfjeld Roberts P, Palmer AJ, et al. The urgent need for evidence in arthroscopic meniscal surgery: a systematic review of the evidence for operative management of meniscal tears. Am J Sports Med. 2017;45:965-973.
8. Sihvonen R, Paavola M, Malmivaara A, et al; Finnish Degenerative Meniscal Lesion Study (FIDELITY) Group. Arthroscopic partial meniscectomy versus sham surgery for a degenerative meniscal tear. N Engl J Med. 2013;369:2515-2524.
9. Beaufils P, Hulet C, Dhénain M, et al. Clinical practice guidelines for the management of meniscal lesions and isolated lesions of the anterior cruciate ligament of the knee in adults. Orthop Traumatol Surg Res. 2009;95:437-442.

Article PDF
Author and Disclosure Information

Jennie B. Jarrett is with the University of Illinois at Chicago. Sandra Sauereisen is with the St. Margaret Family Medicine Residency Program at the University of Pittsburgh Medical Center.

Issue
Clinician Reviews - 27(5)
Publications
Topics
Page Number
14-16
Sections
Author and Disclosure Information

Jennie B. Jarrett is with the University of Illinois at Chicago. Sandra Sauereisen is with the St. Margaret Family Medicine Residency Program at the University of Pittsburgh Medical Center.

Author and Disclosure Information

Jennie B. Jarrett is with the University of Illinois at Chicago. Sandra Sauereisen is with the St. Margaret Family Medicine Residency Program at the University of Pittsburgh Medical Center.

Article PDF
Article PDF
Related Articles

 

A 48-year-old man presents to your office for follow-up of right knee pain that has been bothering him for the past 12 months. He denies any trauma or inciting incident for the pain. On physical exam, he does not have crepitus but does have medial joint line tenderness of his right knee. An MRI shows a partial medial meniscal tear. Do you refer him to physical therapy (PT) or to orthopedics for arthroscopy and repair?

The meniscus—cartilage in the knee joint that provides support, stability, and lubrication to the joint during activity—can tear during a traumatic event or as a result of degeneration over time. Traumatic meniscal tears typically occur in those younger than 30 during sports (eg, basketball, soccer), whereas degenerative meniscal tears generally occur in patients ages 40 to 60.2,3 The annual incidence of all meniscal tears is 79 per 100,000.4 While some clinicians can diagnose traumatic meniscal tears based on history and physical examination, degenerative meniscal tears are more challenging and typically warrant an MRI for confirmation.3

Meniscal tears can be treated either conservatively, with supportive care and exercise, or surgically. Unfortunately, there are no national orthopedic guidelines available to help direct care. In one observational study, 95 of 117 patients (81.2%) were generally satisfied with surgical treatment at four-year follow-up; satisfaction was higher among those with a traumatic meniscal tear than in those with a degenerative tear.5

Two systematic reviews of surgery versus nonoperative management or sham therapies found no additional benefit of surgery for meniscal tears in a variety of patients with and without osteoarthritis.6,7 However, both studies were of only moderate quality, because of the number of patients in the nonoperative groups who ultimately underwent surgery. Neither of the studies directly compared surgery to nonoperative management.6,7Another investigation—a multicenter, randomized, double-blind, sham-controlled study conducted in Finland involving 146 patients—compared sham surgery to arthroscopic partial meniscectomy. Both groups received instruction on performing post-procedure exercises, and both groups had similar and marked improvement in pain and function.8

Clinical practice recommendations devised from a vast systematic review of the literature recommend that the decision for surgery be based on patient-specific factors, such as symptoms, age, mechanism of tear, extent of damage, and occupational/social/activity needs.9

STUDY SUMMARY

Exercise is as good as surgery

The current superiority RCT compared exercise therapy to arthroscopic partial meniscectomy. Subjects (ages 35 to 60) presented to the orthopedic department of two hospitals in Norway with unilateral knee pain of more than two months’ duration and an MRI-delineated medial meniscal tear. They were included in the study only if they had radiographic evidence of minimal osteoarthritis (Kellgren-Lawrence classification grade ≤ 2). Exclusion criteria included acute trauma, locked knee, ligament injury, and knee surgery in the same knee within the previous two years.

The primary outcomes were change in patient-reported knee function (as determined by overall Knee injury and Osteoarthritis Outcome Score [KOOS] after two years) and thigh muscle strength at three months (as measured by physiotherapists). The researchers used four of the five KOOS subscales for this analysis: pain, other symptoms (swelling, grinding/noise from the joint, ability to straighten and bend), function in sports/recreation, and knee-related quality of life (QOL). The average score of each subscale was used.

Secondary outcomes included the five individual KOOS subscales (the four previously mentioned, plus activities of daily living [ADLs]), as well as thigh muscle strength and lower-extremity performance test results.

Methods. Testing personnel were blinded to group allocation; participants wore pants or neoprene sleeves to cover surgical scars. A total of 140 patients were randomized to either 12 weeks (24-36 sessions) of exercise therapy alone or a standardized arthroscopic partial meniscectomy; upon discharge, those in the latter group received written and oral encouragement to perform simple exercises at home, two to four times daily, to regain range of motion and reduce swelling.

Results. At two years, the overall mean improvement in KOOS4 score from baseline was similar between the exercise group and the meniscectomy group (25.3 pts vs 24.4 pts, respectively; mean difference [MD], 0.9). Additionally, muscle strength (measured as peak torque flexion and extension and total work flexion and extension) at both three and 12 months showed significant objective improvements favoring exercise therapy.

In the secondary analysis of the KOOS subscale scores, change from baseline was nonsignificant for four of the five (pain, ADL, sports/recreation, and QOL). Only the symptoms subscale had a significant difference favoring exercise therapy (MD, 5.3 pts); this was likely clinically insignificant on a grading scale of 0 to 100.

Of the patients allocated to exercise therapy alone, 19% crossed over and underwent surgery during the two-year study period.

 

 

 

WHAT’S NEW

Head-to-head comparison adds evidence

This is the first trial to directly compare exercise therapy to surgery in patients with meniscal tears. Interestingly, exercise therapy was as effective after a two-year follow-up period and was superior in the short term for thigh muscle strength.1

The results of this study build on those from the aforementioned smaller study conducted in Finland.8 In that study, both groups received instruction for the same graduated exercise plan. The researchers found that exercise was comparable to surgery for meniscal tears in patients with no osteoarthritis.

CAVEATS

What about more severe osteoarthritis?

This trial included patients with no to mild osteoarthritis in addition to their meniscal tear.1 It is unclear if the results would be maintained in those with more advanced disease. Additionally, 19% of patients crossed over from the exercise group to the surgery group, even though muscle strength improved. Therefore, education about the risks of surgery and the potential lack of benefit is important.

CHALLENGES TO IMPLEMENTATION

Cost and effort of PT

The cost of PT can be a barrier for patients who have adequate insurance coverage for surgery but inadequate coverage for PT. Additionally, exercise therapy requires significant and ongoing time and effort, which may deter those with busy lifestyles. Patients and clinicians may view surgery as an “easier” fix.

ACKNOWLEDGEMENT

The PURLs Surveillance System was supported in part by Grant Number UL1RR024999 from the National Center For Research Resources, a Clinical Translational Science Award to the University of Chicago. The content is solely the responsibility of the authors and does not necessarily represent the official views of the National Center For Research Resources or the National Institutes of Health.

Copyright © 2017. The Family Physicians Inquiries Network. All rights reserved.

Reprinted with permission from the Family Physicians Inquiries Network and The Journal of Family Practice (2017;66[4]:250-252).

 

A 48-year-old man presents to your office for follow-up of right knee pain that has been bothering him for the past 12 months. He denies any trauma or inciting incident for the pain. On physical exam, he does not have crepitus but does have medial joint line tenderness of his right knee. An MRI shows a partial medial meniscal tear. Do you refer him to physical therapy (PT) or to orthopedics for arthroscopy and repair?

The meniscus—cartilage in the knee joint that provides support, stability, and lubrication to the joint during activity—can tear during a traumatic event or as a result of degeneration over time. Traumatic meniscal tears typically occur in those younger than 30 during sports (eg, basketball, soccer), whereas degenerative meniscal tears generally occur in patients ages 40 to 60.2,3 The annual incidence of all meniscal tears is 79 per 100,000.4 While some clinicians can diagnose traumatic meniscal tears based on history and physical examination, degenerative meniscal tears are more challenging and typically warrant an MRI for confirmation.3

Meniscal tears can be treated either conservatively, with supportive care and exercise, or surgically. Unfortunately, there are no national orthopedic guidelines available to help direct care. In one observational study, 95 of 117 patients (81.2%) were generally satisfied with surgical treatment at four-year follow-up; satisfaction was higher among those with a traumatic meniscal tear than in those with a degenerative tear.5

Two systematic reviews of surgery versus nonoperative management or sham therapies found no additional benefit of surgery for meniscal tears in a variety of patients with and without osteoarthritis.6,7 However, both studies were of only moderate quality, because of the number of patients in the nonoperative groups who ultimately underwent surgery. Neither of the studies directly compared surgery to nonoperative management.6,7Another investigation—a multicenter, randomized, double-blind, sham-controlled study conducted in Finland involving 146 patients—compared sham surgery to arthroscopic partial meniscectomy. Both groups received instruction on performing post-procedure exercises, and both groups had similar and marked improvement in pain and function.8

Clinical practice recommendations devised from a vast systematic review of the literature recommend that the decision for surgery be based on patient-specific factors, such as symptoms, age, mechanism of tear, extent of damage, and occupational/social/activity needs.9

STUDY SUMMARY

Exercise is as good as surgery

The current superiority RCT compared exercise therapy to arthroscopic partial meniscectomy. Subjects (ages 35 to 60) presented to the orthopedic department of two hospitals in Norway with unilateral knee pain of more than two months’ duration and an MRI-delineated medial meniscal tear. They were included in the study only if they had radiographic evidence of minimal osteoarthritis (Kellgren-Lawrence classification grade ≤ 2). Exclusion criteria included acute trauma, locked knee, ligament injury, and knee surgery in the same knee within the previous two years.

The primary outcomes were change in patient-reported knee function (as determined by overall Knee injury and Osteoarthritis Outcome Score [KOOS] after two years) and thigh muscle strength at three months (as measured by physiotherapists). The researchers used four of the five KOOS subscales for this analysis: pain, other symptoms (swelling, grinding/noise from the joint, ability to straighten and bend), function in sports/recreation, and knee-related quality of life (QOL). The average score of each subscale was used.

Secondary outcomes included the five individual KOOS subscales (the four previously mentioned, plus activities of daily living [ADLs]), as well as thigh muscle strength and lower-extremity performance test results.

Methods. Testing personnel were blinded to group allocation; participants wore pants or neoprene sleeves to cover surgical scars. A total of 140 patients were randomized to either 12 weeks (24-36 sessions) of exercise therapy alone or a standardized arthroscopic partial meniscectomy; upon discharge, those in the latter group received written and oral encouragement to perform simple exercises at home, two to four times daily, to regain range of motion and reduce swelling.

Results. At two years, the overall mean improvement in KOOS4 score from baseline was similar between the exercise group and the meniscectomy group (25.3 pts vs 24.4 pts, respectively; mean difference [MD], 0.9). Additionally, muscle strength (measured as peak torque flexion and extension and total work flexion and extension) at both three and 12 months showed significant objective improvements favoring exercise therapy.

In the secondary analysis of the KOOS subscale scores, change from baseline was nonsignificant for four of the five (pain, ADL, sports/recreation, and QOL). Only the symptoms subscale had a significant difference favoring exercise therapy (MD, 5.3 pts); this was likely clinically insignificant on a grading scale of 0 to 100.

Of the patients allocated to exercise therapy alone, 19% crossed over and underwent surgery during the two-year study period.

 

 

 

WHAT’S NEW

Head-to-head comparison adds evidence

This is the first trial to directly compare exercise therapy to surgery in patients with meniscal tears. Interestingly, exercise therapy was as effective after a two-year follow-up period and was superior in the short term for thigh muscle strength.1

The results of this study build on those from the aforementioned smaller study conducted in Finland.8 In that study, both groups received instruction for the same graduated exercise plan. The researchers found that exercise was comparable to surgery for meniscal tears in patients with no osteoarthritis.

CAVEATS

What about more severe osteoarthritis?

This trial included patients with no to mild osteoarthritis in addition to their meniscal tear.1 It is unclear if the results would be maintained in those with more advanced disease. Additionally, 19% of patients crossed over from the exercise group to the surgery group, even though muscle strength improved. Therefore, education about the risks of surgery and the potential lack of benefit is important.

CHALLENGES TO IMPLEMENTATION

Cost and effort of PT

The cost of PT can be a barrier for patients who have adequate insurance coverage for surgery but inadequate coverage for PT. Additionally, exercise therapy requires significant and ongoing time and effort, which may deter those with busy lifestyles. Patients and clinicians may view surgery as an “easier” fix.

ACKNOWLEDGEMENT

The PURLs Surveillance System was supported in part by Grant Number UL1RR024999 from the National Center For Research Resources, a Clinical Translational Science Award to the University of Chicago. The content is solely the responsibility of the authors and does not necessarily represent the official views of the National Center For Research Resources or the National Institutes of Health.

Copyright © 2017. The Family Physicians Inquiries Network. All rights reserved.

Reprinted with permission from the Family Physicians Inquiries Network and The Journal of Family Practice (2017;66[4]:250-252).

References

1. Kise NJ, Risberg MA, Stensrud S, et al. Exercise therapy versus arthroscopic partial meniscectomy for degenerative meniscal tear in middle aged patients: randomised controlled trial with two year follow-up. BMJ. 2016;354:i3740.
2. Beals CT, Magnussen RA, Graham WC, et al. The prevalence of meniscal pathology in asymptomatic athletes. Sports Med. 2016;46:1517-1524.
3. Maffulli N, Longo UG, Campi S, et al. Meniscal tears. Open Access J Sports Med. 2010;1:45-54.
4. Peat G, Bergknut C, Frobell R, et al. Population-wide incidence estimates for soft tissue knee injuries presenting to healthcare in southern Sweden: data from the Skåne Healthcare Register. Arthritis Res Ther. 2014;16:R162.
5. Ghislain NA, Wei JN, Li YG. Study of the clinical outcome between traumatic and degenerative (non-traumatic) meniscal tears after arthroscopic surgery: a 4-years follow-up study. J Clin Diagn Res. 2016;10:RC01-RC04.
6. Khan M, Evaniew N, Bedi A, et al. Arthroscopic surgery for degenerative tears of the meniscus: a systematic review and meta-analysis. CMAJ. 2014;186:1057-1064.
7. Monk P, Garfjeld Roberts P, Palmer AJ, et al. The urgent need for evidence in arthroscopic meniscal surgery: a systematic review of the evidence for operative management of meniscal tears. Am J Sports Med. 2017;45:965-973.
8. Sihvonen R, Paavola M, Malmivaara A, et al; Finnish Degenerative Meniscal Lesion Study (FIDELITY) Group. Arthroscopic partial meniscectomy versus sham surgery for a degenerative meniscal tear. N Engl J Med. 2013;369:2515-2524.
9. Beaufils P, Hulet C, Dhénain M, et al. Clinical practice guidelines for the management of meniscal lesions and isolated lesions of the anterior cruciate ligament of the knee in adults. Orthop Traumatol Surg Res. 2009;95:437-442.

References

1. Kise NJ, Risberg MA, Stensrud S, et al. Exercise therapy versus arthroscopic partial meniscectomy for degenerative meniscal tear in middle aged patients: randomised controlled trial with two year follow-up. BMJ. 2016;354:i3740.
2. Beals CT, Magnussen RA, Graham WC, et al. The prevalence of meniscal pathology in asymptomatic athletes. Sports Med. 2016;46:1517-1524.
3. Maffulli N, Longo UG, Campi S, et al. Meniscal tears. Open Access J Sports Med. 2010;1:45-54.
4. Peat G, Bergknut C, Frobell R, et al. Population-wide incidence estimates for soft tissue knee injuries presenting to healthcare in southern Sweden: data from the Skåne Healthcare Register. Arthritis Res Ther. 2014;16:R162.
5. Ghislain NA, Wei JN, Li YG. Study of the clinical outcome between traumatic and degenerative (non-traumatic) meniscal tears after arthroscopic surgery: a 4-years follow-up study. J Clin Diagn Res. 2016;10:RC01-RC04.
6. Khan M, Evaniew N, Bedi A, et al. Arthroscopic surgery for degenerative tears of the meniscus: a systematic review and meta-analysis. CMAJ. 2014;186:1057-1064.
7. Monk P, Garfjeld Roberts P, Palmer AJ, et al. The urgent need for evidence in arthroscopic meniscal surgery: a systematic review of the evidence for operative management of meniscal tears. Am J Sports Med. 2017;45:965-973.
8. Sihvonen R, Paavola M, Malmivaara A, et al; Finnish Degenerative Meniscal Lesion Study (FIDELITY) Group. Arthroscopic partial meniscectomy versus sham surgery for a degenerative meniscal tear. N Engl J Med. 2013;369:2515-2524.
9. Beaufils P, Hulet C, Dhénain M, et al. Clinical practice guidelines for the management of meniscal lesions and isolated lesions of the anterior cruciate ligament of the knee in adults. Orthop Traumatol Surg Res. 2009;95:437-442.

Issue
Clinician Reviews - 27(5)
Issue
Clinician Reviews - 27(5)
Page Number
14-16
Page Number
14-16
Publications
Publications
Topics
Article Type
Display Headline
When Can Exercise Supplant Surgery for Degenerative Meniscal Tears?
Display Headline
When Can Exercise Supplant Surgery for Degenerative Meniscal Tears?
Sections
Disallow All Ads
Content Gating
No Gating (article Unlocked/Free)
Alternative CME
Article PDF Media

Consider melatonin for migraine prevention

Article Type
Changed
Tue, 06/23/2020 - 13:35
Display Headline
Consider melatonin for migraine prevention
 

ILLUSTRATIVE CASE

A 32-year-old woman comes to your office for help with her recurrent migraines, which she’s had since her early 20s. She is otherwise healthy and active. She is frustrated over the frequency of her migraines and the debilitation they cause. She has tried prophylactic medications in the past, but stopped taking them because of the adverse effects. What do you recommend for treatment?

Daily preventive medication can be helpful for chronic migraine sufferers whose headaches have a significant impact on their lives and who have a goal of reducing headache frequency or severity, disability, and/or avoiding acute headache medication escalation.2 An estimated 38% of patients with migraines are appropriate candidates for prophylactic therapy, but only 3% to 13% are taking preventive medications.3

Evidence-based guidelines from the American Academy of Neurology and the American Headache Society state that antiepileptic drugs (divalproex sodium, sodium valproate, topiramate) and many beta-blockers (metoprolol, propranolol, timolol) are effective and should be recommended for migraine prevention (level A recommendation; based on ≥2 class I trials).2 Medications such as antidepressants (amitriptyline, venlafaxine) and other beta-blockers (atenolol, nadolol) are probably effective and can be considered (level B recommendation; based on one class I trial or 2 class II trials).2 However, adverse effects, such as somnolence, are listed as frequent with amitriptyline and occasional to frequent with topiramate.4

Researchers have investigated melatonin before. But a 2010 double-blind, crossover, randomized controlled trial (RCT) of 46 patients with 2 to 7 migraine attacks per month found no significant difference in reduction of headache frequency with extended-release melatonin 2 mg taken one hour before bed compared to placebo over an 8-week period.5

[polldaddy:9724288]

STUDY SUMMARY

Melatonin tops amitriptyline in >50% improvement in headache frequency

This RCT conducted in Brazil compared the effectiveness of melatonin to amitriptyline and placebo for migraine prevention in 196 adults (ages 18-65 years) with chronic migraines.1 Eligible patients had a history of at least 3 migraine attacks or 4 migraine headache days per month. Patients were randomized to take identically-appearing melatonin 3 mg, amitriptyline 25 mg, or placebo nightly. The investigators appear to have concealed allocation adequately, and used double-blinding.

The primary outcome was the number of headache days per month, comparing baseline with the 4 weeks of treatment. Secondary endpoints included reduction in migraine intensity, duration, number of analgesics used, and percentage of patients with more than 50% reduction in migraine headache days.

An estimated 38% of patients with migraines are appropriate candidates for prophylactic therapy, but only 3% to 13% are taking preventive medications.

Compared to placebo, headache days per month were reduced in both the melatonin group (6.2 days vs 4.6 days, respectively; mean difference [MD], -1.6; 95% confidence interval [CI], -2.4 to -0.9) and the amitriptyline group (6.2 days vs 5 days, respectively; MD, -1.1; 95% CI, -1.5 to -0.7) at 12 weeks, based on intention-to-treat analysis. Mean headache intensity (0-10 pain scale) was also lower at 12 weeks in the melatonin group (4.8 vs 3.6; MD, -1.2; 95% CI, -1.6 to -0.8) and in the amitriptyline group (4.8 vs 3.5; MD, -1.3; 95% CI, -1.7 to -0.9), when compared to placebo.

Headache duration (hours/month) at 12 weeks was reduced in both groups (amitriptyline MD, -4.4 hours; 95% CI, -5.1 to -3.9; melatonin MD, -4.8 hours; 95% CI, -5.7 to -3.9), as was the number of analgesics used (amitriptyline MD, -1; 95% CI, -1.5 to -0.5; melatonin MD, -1; 95% CI, -1.4 to -0.6) when compared to placebo. There was no significant difference between the melatonin and amitriptyline groups for these outcomes.

Patients taking melatonin were more likely to have a >50% improvement in headache frequency compared to amitriptyline (54% vs 39%; number needed to treat [NNT]=7; P<.05); melatonin worked much better than placebo (54% vs 20%; NNT=3; P<.01).

Adverse events were reported more often in the amitriptyline group than in the melatonin group (46 vs 16; P<.03) with daytime sleepiness being the most frequent complaint (41% of patients in the amitriptyline group vs 18% of the melatonin group; number needed to harm [NNH]=5). There was no significant difference in adverse events between melatonin and placebo (16 vs 17; P=not significant). Melatonin resulted in weight loss (mean, -0.14 kg), whereas those taking amitriptyline gained weight (+0.97 kg; P<.01).

 

 

 

WHAT’S NEW

An effective migraine prevention alternative with minimal adverse effects

Melatonin is an accessible and affordable option for preventing migraine headaches in chronic sufferers. The 3-mg dosing reduces headache frequency—both in terms of the number of migraine headache days per month and in terms of the percentage of patients with a >50% reduction in headache events—as well as headache intensity, with minimal adverse effects.

CAVEATS

Product consistency, missing study data

This trial used 3-mg dosing, so it is not clear if other doses are also effective. In addition, because melatonin is available over-the-counter, the quality/actual doses may be less well regulated, and thus, there may be a lack of consistency between brands. Unlike clinical practice, neither the amitriptyline nor the melatonin dose was titrated according to patient response or adverse effects. As a result, we are not sure of the actual lowest effective dose, or if greater effect (with continued minimal adverse effects) could be achieved with higher doses.

Lastly, 69% to 75% of patients in the treatment groups completed the 16-week trial, but the authors of the study reported using 3 different analytic techniques to estimate missing data. The primary outcome included 178 of 196 randomized patients (90.8%). For the primary endpoint, the authors treated all missing data as non-headache days. It is unclear how these missing data would affect the outcome, although an analysis like this would tend towards a null effect.

CHALLENGES TO IMPLEMENTATION

Challenges are negligible

There are really no challenges to implementing this practice changer; melatonin is readily available over-the-counter and it is affordable.

ACKNOWLEDGEMENT

The PURLs Surveillance System was supported in part by Grant Number UL1RR024999 from the National Center For Research Resources, a Clinical Translational Science Award to the University of Chicago. The content is solely the responsibility of the authors and does not necessarily represent the official views of the National Center For Research Resources or the National Institutes of Health.

Files
References

1. Gonçalves AL, Martini Ferreira A, Ribeiro RT, et al. Randomised clinical trial comparing melatonin 3 mg, amitriptyline 25 mg and placebo for migraine prevention. J Neurol Neurosurg Psychiatry. 2016;87:1127-1132.

2. Silberstein SD, Holland S, Freitag F, et al. Evidence-based guideline update: pharmacologic treatment for episodic migraine prevention in adults: report of the Quality Standards Subcommittee of the American Academy of Neurology and the American Headache Society. Neurology. 2012;78:1337-1345.

3. Lipton RB, Bigal ME, Diamond M, et al; The American Migraine Prevalence and Prevention Advisory Group. Migraine prevalence, disease burden, and the need for preventive therapy. Neurology. 2007;68:343-349.

4. Silberstein SD. Practice parameter: evidence-based guidelines for migraine headache (an evidence-based review): report of the Quality Standards Subcommittee of the American Academy of Neurology. Neurology. 2000;55:754-762.

5. Alstadhaug KB, Odeh F, Salvesen R, et al. Prophylaxis of migraine with melatonin: a randomized controlled trial. Neurology. 2010;75:1527-1532.

Article PDF
Author and Disclosure Information

University of Colorado Family Medicine Residency, Denver

DEPUTY EDITOR
James J. Stevermer, MD, MSPH

Department of Family and Community Medicine, University of Missouri-Columbia

Issue
The Journal of Family Practice - 66(5)
Publications
Topics
Page Number
320-322
Sections
Files
Files
Author and Disclosure Information

University of Colorado Family Medicine Residency, Denver

DEPUTY EDITOR
James J. Stevermer, MD, MSPH

Department of Family and Community Medicine, University of Missouri-Columbia

Author and Disclosure Information

University of Colorado Family Medicine Residency, Denver

DEPUTY EDITOR
James J. Stevermer, MD, MSPH

Department of Family and Community Medicine, University of Missouri-Columbia

Article PDF
Article PDF
 

ILLUSTRATIVE CASE

A 32-year-old woman comes to your office for help with her recurrent migraines, which she’s had since her early 20s. She is otherwise healthy and active. She is frustrated over the frequency of her migraines and the debilitation they cause. She has tried prophylactic medications in the past, but stopped taking them because of the adverse effects. What do you recommend for treatment?

Daily preventive medication can be helpful for chronic migraine sufferers whose headaches have a significant impact on their lives and who have a goal of reducing headache frequency or severity, disability, and/or avoiding acute headache medication escalation.2 An estimated 38% of patients with migraines are appropriate candidates for prophylactic therapy, but only 3% to 13% are taking preventive medications.3

Evidence-based guidelines from the American Academy of Neurology and the American Headache Society state that antiepileptic drugs (divalproex sodium, sodium valproate, topiramate) and many beta-blockers (metoprolol, propranolol, timolol) are effective and should be recommended for migraine prevention (level A recommendation; based on ≥2 class I trials).2 Medications such as antidepressants (amitriptyline, venlafaxine) and other beta-blockers (atenolol, nadolol) are probably effective and can be considered (level B recommendation; based on one class I trial or 2 class II trials).2 However, adverse effects, such as somnolence, are listed as frequent with amitriptyline and occasional to frequent with topiramate.4

Researchers have investigated melatonin before. But a 2010 double-blind, crossover, randomized controlled trial (RCT) of 46 patients with 2 to 7 migraine attacks per month found no significant difference in reduction of headache frequency with extended-release melatonin 2 mg taken one hour before bed compared to placebo over an 8-week period.5

[polldaddy:9724288]

STUDY SUMMARY

Melatonin tops amitriptyline in >50% improvement in headache frequency

This RCT conducted in Brazil compared the effectiveness of melatonin to amitriptyline and placebo for migraine prevention in 196 adults (ages 18-65 years) with chronic migraines.1 Eligible patients had a history of at least 3 migraine attacks or 4 migraine headache days per month. Patients were randomized to take identically-appearing melatonin 3 mg, amitriptyline 25 mg, or placebo nightly. The investigators appear to have concealed allocation adequately, and used double-blinding.

The primary outcome was the number of headache days per month, comparing baseline with the 4 weeks of treatment. Secondary endpoints included reduction in migraine intensity, duration, number of analgesics used, and percentage of patients with more than 50% reduction in migraine headache days.

An estimated 38% of patients with migraines are appropriate candidates for prophylactic therapy, but only 3% to 13% are taking preventive medications.

Compared to placebo, headache days per month were reduced in both the melatonin group (6.2 days vs 4.6 days, respectively; mean difference [MD], -1.6; 95% confidence interval [CI], -2.4 to -0.9) and the amitriptyline group (6.2 days vs 5 days, respectively; MD, -1.1; 95% CI, -1.5 to -0.7) at 12 weeks, based on intention-to-treat analysis. Mean headache intensity (0-10 pain scale) was also lower at 12 weeks in the melatonin group (4.8 vs 3.6; MD, -1.2; 95% CI, -1.6 to -0.8) and in the amitriptyline group (4.8 vs 3.5; MD, -1.3; 95% CI, -1.7 to -0.9), when compared to placebo.

Headache duration (hours/month) at 12 weeks was reduced in both groups (amitriptyline MD, -4.4 hours; 95% CI, -5.1 to -3.9; melatonin MD, -4.8 hours; 95% CI, -5.7 to -3.9), as was the number of analgesics used (amitriptyline MD, -1; 95% CI, -1.5 to -0.5; melatonin MD, -1; 95% CI, -1.4 to -0.6) when compared to placebo. There was no significant difference between the melatonin and amitriptyline groups for these outcomes.

Patients taking melatonin were more likely to have a >50% improvement in headache frequency compared to amitriptyline (54% vs 39%; number needed to treat [NNT]=7; P<.05); melatonin worked much better than placebo (54% vs 20%; NNT=3; P<.01).

Adverse events were reported more often in the amitriptyline group than in the melatonin group (46 vs 16; P<.03) with daytime sleepiness being the most frequent complaint (41% of patients in the amitriptyline group vs 18% of the melatonin group; number needed to harm [NNH]=5). There was no significant difference in adverse events between melatonin and placebo (16 vs 17; P=not significant). Melatonin resulted in weight loss (mean, -0.14 kg), whereas those taking amitriptyline gained weight (+0.97 kg; P<.01).

 

 

 

WHAT’S NEW

An effective migraine prevention alternative with minimal adverse effects

Melatonin is an accessible and affordable option for preventing migraine headaches in chronic sufferers. The 3-mg dosing reduces headache frequency—both in terms of the number of migraine headache days per month and in terms of the percentage of patients with a >50% reduction in headache events—as well as headache intensity, with minimal adverse effects.

CAVEATS

Product consistency, missing study data

This trial used 3-mg dosing, so it is not clear if other doses are also effective. In addition, because melatonin is available over-the-counter, the quality/actual doses may be less well regulated, and thus, there may be a lack of consistency between brands. Unlike clinical practice, neither the amitriptyline nor the melatonin dose was titrated according to patient response or adverse effects. As a result, we are not sure of the actual lowest effective dose, or if greater effect (with continued minimal adverse effects) could be achieved with higher doses.

Lastly, 69% to 75% of patients in the treatment groups completed the 16-week trial, but the authors of the study reported using 3 different analytic techniques to estimate missing data. The primary outcome included 178 of 196 randomized patients (90.8%). For the primary endpoint, the authors treated all missing data as non-headache days. It is unclear how these missing data would affect the outcome, although an analysis like this would tend towards a null effect.

CHALLENGES TO IMPLEMENTATION

Challenges are negligible

There are really no challenges to implementing this practice changer; melatonin is readily available over-the-counter and it is affordable.

ACKNOWLEDGEMENT

The PURLs Surveillance System was supported in part by Grant Number UL1RR024999 from the National Center For Research Resources, a Clinical Translational Science Award to the University of Chicago. The content is solely the responsibility of the authors and does not necessarily represent the official views of the National Center For Research Resources or the National Institutes of Health.

 

ILLUSTRATIVE CASE

A 32-year-old woman comes to your office for help with her recurrent migraines, which she’s had since her early 20s. She is otherwise healthy and active. She is frustrated over the frequency of her migraines and the debilitation they cause. She has tried prophylactic medications in the past, but stopped taking them because of the adverse effects. What do you recommend for treatment?

Daily preventive medication can be helpful for chronic migraine sufferers whose headaches have a significant impact on their lives and who have a goal of reducing headache frequency or severity, disability, and/or avoiding acute headache medication escalation.2 An estimated 38% of patients with migraines are appropriate candidates for prophylactic therapy, but only 3% to 13% are taking preventive medications.3

Evidence-based guidelines from the American Academy of Neurology and the American Headache Society state that antiepileptic drugs (divalproex sodium, sodium valproate, topiramate) and many beta-blockers (metoprolol, propranolol, timolol) are effective and should be recommended for migraine prevention (level A recommendation; based on ≥2 class I trials).2 Medications such as antidepressants (amitriptyline, venlafaxine) and other beta-blockers (atenolol, nadolol) are probably effective and can be considered (level B recommendation; based on one class I trial or 2 class II trials).2 However, adverse effects, such as somnolence, are listed as frequent with amitriptyline and occasional to frequent with topiramate.4

Researchers have investigated melatonin before. But a 2010 double-blind, crossover, randomized controlled trial (RCT) of 46 patients with 2 to 7 migraine attacks per month found no significant difference in reduction of headache frequency with extended-release melatonin 2 mg taken one hour before bed compared to placebo over an 8-week period.5

[polldaddy:9724288]

STUDY SUMMARY

Melatonin tops amitriptyline in >50% improvement in headache frequency

This RCT conducted in Brazil compared the effectiveness of melatonin to amitriptyline and placebo for migraine prevention in 196 adults (ages 18-65 years) with chronic migraines.1 Eligible patients had a history of at least 3 migraine attacks or 4 migraine headache days per month. Patients were randomized to take identically-appearing melatonin 3 mg, amitriptyline 25 mg, or placebo nightly. The investigators appear to have concealed allocation adequately, and used double-blinding.

The primary outcome was the number of headache days per month, comparing baseline with the 4 weeks of treatment. Secondary endpoints included reduction in migraine intensity, duration, number of analgesics used, and percentage of patients with more than 50% reduction in migraine headache days.

An estimated 38% of patients with migraines are appropriate candidates for prophylactic therapy, but only 3% to 13% are taking preventive medications.

Compared to placebo, headache days per month were reduced in both the melatonin group (6.2 days vs 4.6 days, respectively; mean difference [MD], -1.6; 95% confidence interval [CI], -2.4 to -0.9) and the amitriptyline group (6.2 days vs 5 days, respectively; MD, -1.1; 95% CI, -1.5 to -0.7) at 12 weeks, based on intention-to-treat analysis. Mean headache intensity (0-10 pain scale) was also lower at 12 weeks in the melatonin group (4.8 vs 3.6; MD, -1.2; 95% CI, -1.6 to -0.8) and in the amitriptyline group (4.8 vs 3.5; MD, -1.3; 95% CI, -1.7 to -0.9), when compared to placebo.

Headache duration (hours/month) at 12 weeks was reduced in both groups (amitriptyline MD, -4.4 hours; 95% CI, -5.1 to -3.9; melatonin MD, -4.8 hours; 95% CI, -5.7 to -3.9), as was the number of analgesics used (amitriptyline MD, -1; 95% CI, -1.5 to -0.5; melatonin MD, -1; 95% CI, -1.4 to -0.6) when compared to placebo. There was no significant difference between the melatonin and amitriptyline groups for these outcomes.

Patients taking melatonin were more likely to have a >50% improvement in headache frequency compared to amitriptyline (54% vs 39%; number needed to treat [NNT]=7; P<.05); melatonin worked much better than placebo (54% vs 20%; NNT=3; P<.01).

Adverse events were reported more often in the amitriptyline group than in the melatonin group (46 vs 16; P<.03) with daytime sleepiness being the most frequent complaint (41% of patients in the amitriptyline group vs 18% of the melatonin group; number needed to harm [NNH]=5). There was no significant difference in adverse events between melatonin and placebo (16 vs 17; P=not significant). Melatonin resulted in weight loss (mean, -0.14 kg), whereas those taking amitriptyline gained weight (+0.97 kg; P<.01).

 

 

 

WHAT’S NEW

An effective migraine prevention alternative with minimal adverse effects

Melatonin is an accessible and affordable option for preventing migraine headaches in chronic sufferers. The 3-mg dosing reduces headache frequency—both in terms of the number of migraine headache days per month and in terms of the percentage of patients with a >50% reduction in headache events—as well as headache intensity, with minimal adverse effects.

CAVEATS

Product consistency, missing study data

This trial used 3-mg dosing, so it is not clear if other doses are also effective. In addition, because melatonin is available over-the-counter, the quality/actual doses may be less well regulated, and thus, there may be a lack of consistency between brands. Unlike clinical practice, neither the amitriptyline nor the melatonin dose was titrated according to patient response or adverse effects. As a result, we are not sure of the actual lowest effective dose, or if greater effect (with continued minimal adverse effects) could be achieved with higher doses.

Lastly, 69% to 75% of patients in the treatment groups completed the 16-week trial, but the authors of the study reported using 3 different analytic techniques to estimate missing data. The primary outcome included 178 of 196 randomized patients (90.8%). For the primary endpoint, the authors treated all missing data as non-headache days. It is unclear how these missing data would affect the outcome, although an analysis like this would tend towards a null effect.

CHALLENGES TO IMPLEMENTATION

Challenges are negligible

There are really no challenges to implementing this practice changer; melatonin is readily available over-the-counter and it is affordable.

ACKNOWLEDGEMENT

The PURLs Surveillance System was supported in part by Grant Number UL1RR024999 from the National Center For Research Resources, a Clinical Translational Science Award to the University of Chicago. The content is solely the responsibility of the authors and does not necessarily represent the official views of the National Center For Research Resources or the National Institutes of Health.

References

1. Gonçalves AL, Martini Ferreira A, Ribeiro RT, et al. Randomised clinical trial comparing melatonin 3 mg, amitriptyline 25 mg and placebo for migraine prevention. J Neurol Neurosurg Psychiatry. 2016;87:1127-1132.

2. Silberstein SD, Holland S, Freitag F, et al. Evidence-based guideline update: pharmacologic treatment for episodic migraine prevention in adults: report of the Quality Standards Subcommittee of the American Academy of Neurology and the American Headache Society. Neurology. 2012;78:1337-1345.

3. Lipton RB, Bigal ME, Diamond M, et al; The American Migraine Prevalence and Prevention Advisory Group. Migraine prevalence, disease burden, and the need for preventive therapy. Neurology. 2007;68:343-349.

4. Silberstein SD. Practice parameter: evidence-based guidelines for migraine headache (an evidence-based review): report of the Quality Standards Subcommittee of the American Academy of Neurology. Neurology. 2000;55:754-762.

5. Alstadhaug KB, Odeh F, Salvesen R, et al. Prophylaxis of migraine with melatonin: a randomized controlled trial. Neurology. 2010;75:1527-1532.

References

1. Gonçalves AL, Martini Ferreira A, Ribeiro RT, et al. Randomised clinical trial comparing melatonin 3 mg, amitriptyline 25 mg and placebo for migraine prevention. J Neurol Neurosurg Psychiatry. 2016;87:1127-1132.

2. Silberstein SD, Holland S, Freitag F, et al. Evidence-based guideline update: pharmacologic treatment for episodic migraine prevention in adults: report of the Quality Standards Subcommittee of the American Academy of Neurology and the American Headache Society. Neurology. 2012;78:1337-1345.

3. Lipton RB, Bigal ME, Diamond M, et al; The American Migraine Prevalence and Prevention Advisory Group. Migraine prevalence, disease burden, and the need for preventive therapy. Neurology. 2007;68:343-349.

4. Silberstein SD. Practice parameter: evidence-based guidelines for migraine headache (an evidence-based review): report of the Quality Standards Subcommittee of the American Academy of Neurology. Neurology. 2000;55:754-762.

5. Alstadhaug KB, Odeh F, Salvesen R, et al. Prophylaxis of migraine with melatonin: a randomized controlled trial. Neurology. 2010;75:1527-1532.

Issue
The Journal of Family Practice - 66(5)
Issue
The Journal of Family Practice - 66(5)
Page Number
320-322
Page Number
320-322
Publications
Publications
Topics
Article Type
Display Headline
Consider melatonin for migraine prevention
Display Headline
Consider melatonin for migraine prevention
Sections
PURLs Copyright

Copyright © 2017. The Family Physicians Inquiries Network. All rights reserved.

Inside the Article

PRACTICE CHANGER

Recommend nightly melatonin 3 mg to your patients with chronic migraines, as it appears to be as effective as amitriptyline in reducing headaches and causes fewer adverse effects.

STRENGTH OF RECOMMENDATION

B: Based on a single, good quality randomized controlled trial.

Gonçalves AL, Martini Ferreira A, Ribeiro RT, et al. Randomised clinical trial comparing melatonin 3 mg, amitriptyline 25 mg and placebo for migraine prevention. J Neurol Neurosurg Psychiatry. 2016;87:1127-1132.1

Disallow All Ads
Content Gating
No Gating (article Unlocked/Free)
Alternative CME
PubMed ID
28459893
Use ProPublica
Hide sidebar & use full width
render the right sidebar.
Conference Recap Checkbox
Not Conference Recap
Clinical Edge
Article PDF Media
Media Files

“Cold Turkey” Works Best for Smoking Cessation

Article Type
Changed
Thu, 04/06/2017 - 00:16
Display Headline
“Cold Turkey” Works Best for Smoking Cessation

 

A 43-year-old man has a 35–pack-year smoking history and currently smokes one pack of cigarettes a day. He is eager to quit smoking since a close friend of his was recently diagnosed with lung cancer. He asks whether he should quit “cold turkey” or gradually. What do you recommend?

Between 2013 and 2014, one in five American adults reported using tobacco products some days or every day, and 66% of smokers in 2013 made at least one attempt to quit.2,3 The risks of tobacco use and the benefits of cessation are well established, and behavioral and pharmacologic interventions (both alone and in combination) increase smoking cessation rates.4 The US Preventive Services Task Force recommends that health care providers address tobacco use and cessation with patients at regular office visits and offer behavioral and pharmacologic interventions.5 Current guidelines, however, make no specific recommendations regarding gradual versus abrupt smoking cessation methods.5

A previous Cochrane review of 10 RCTs demonstrated no significant difference in quit rates between gradual cigarette reduction and abrupt cessation. The meta-analysis was limited, however, by differences in patient populations, outcome definitions, and types of interventions (both pharmacologic and behavioral).6

In a retrospective cohort study, French investigators reviewed an online database of more than 60,000 smokers who presented to nationwide cessation services. The researchers found that older participants (those 45 and older) and heavy smokers (≥ 21 cigarettes/d) were more likely to quit gradually than abruptly.7

STUDY SUMMARY

“Cold turkey” is better than gradual cessation at six months

A noninferiority RCT was conducted in England to assess whether gradual smoking cessation is as successful as abrupt cessation.1 The primary outcome was abstinence from smoking at four weeks, assessed using the Russell Standard. This set of six criteria (including validation by exhaled CO concentrations of < 10 ppm) is used by the National Centre for Smoking Cessation and Training to decrease variability of reported smoking cessation rates in English studies.8

Participants were recruited via letters from their primary care practice inviting them to participate in a smoking cessation study. The 697 subjects were randomized to either the abrupt-cessation group or the gradual-cessation group. Baseline characteristics were similar between groups.

All participants were asked to schedule a quit date for two weeks after their enrollment. Patients assigned to the gradual-cessation group were provided nicotine replacement patches (21 mg/d) and their choice of short-acting nicotine replacement therapy (NRT; gum, lozenges, nasal spray, sublingual tablets, inhalator, or mouth spray) to use in the two weeks leading up to the quit date. They were given instructions to reduce smoking by half of the baseline amount by the end of the first week, and to a quarter of baseline by the end of the second week.

Patients randomly assigned to the abrupt-cessation group were instructed to continue their current smoking habits until the cessation date; during those two weeks they were given nicotine patches (because the other group received them, and some evidence suggests that precessation NRT increases quit rates) but no short-acting NRT.

Following the cessation date, treatment in both groups was identical, including behavioral support, nicotine patches (21 mg/d), and the patient’s choice of short-acting NRT. Behavioral support consisted of visits with a research nurse at the patient’s primary care practice at the following intervals: weekly for two weeks before the quit date; the day before the quit date; weekly for four weeks after the quit date; and eight weeks after the quit date.

The chosen noninferiority margin was equal to a relative risk (RR) of 0.81 (19% reduction in effectiveness) of quitting gradually, compared with abrupt cessation of smoking. Quit rates in the gradual-reduction group did not reach the threshold for noninferiority; in fact, four-week abstinence was significantly more likely in the abrupt-cessation group than in the gradual-cessation group (49% vs 39.2%; RR, 0.80; number needed to treat [NNT], 10). Similarly, secondary outcomes of eight-week and six-month abstinence rates showed superiority of abrupt over gradual cessation. Six months after the quit date, 15.5% of the gradual-cessation group and 22% of the abrupt-cessation group remained abstinent (RR, 0.71; NNT, 15).

 

 

 

Patient preference plays a role

The investigators also found a difference in successful cessation based on the participants’ preferred method of cessation. Participants who preferred abrupt cessation were more likely to be abstinent at four weeks than participants who preferred gradual cessation (52.2% vs 38.3%).

Patients with a baseline preference for gradual cessation were equally as likely to successfully quit when allocated to abrupt cessation against their preference as when they were allocated to gradual cessation. Four-week abstinence was seen in 34.6% of patients who preferred and were allocated to gradual cessation and in 42% of patients who preferred gradual but were allocated to abrupt cessation.

WHAT’S NEW

Higher quality study; added element of preference

This large, well-designed, noninferiority study showed that abrupt cessation is superior to gradual cessation. The size and design of the study, including a standardized method of assessing cessation and a standardized intervention, make this a higher quality study than those in the Cochrane meta-analysis.6 This study also showed that participants who preferred gradual cessation were less likely to be successful—regardless of the method to which they were assigned.

CAVEATS

Generalizability limited by race and number of cigarettes smoked

Patients lost to follow-up at four weeks (35 in the abrupt-cessation group and 48 in the gradual-cessation group) were assumed to have continued smoking, which may have biased the results toward abrupt cessation. That said, the large number of study participants, along with the relatively small number lost to follow-up, minimizes this weakness.

The majority of participants were white, which may limit generalizability to nonwhite populations. In addition, participants smoked an average of 20 cigarettes per day and, as noted previously, an observational study of tobacco users in France found that heavy smokers (≥ 21 cigarettes/d) were more likely to quit gradually than abruptly. Therefore, results may not be generalizable to heavy smokers.7

CHALLENGES TO IMPLEMENTATION

Considerable investment in behavioral support

One significant challenge is the implementation of such a structured tobacco cessation program in primary care. Both abrupt- and gradual-cessation groups were given considerable behavioral support from research nurses. Participants in this study were seen by a nurse seven times in the first six weeks of the study, and the intervention included nurse-created reduction schedules.

Even if patients in the study preferred one method of cessation to another, they were receptive to quitting either gradually or abruptly. In clinical practice, patients are often set in their desired method of cessation. In that setting, our role is then to inform them of the data and support them in whatever method they choose.

ACKNOWLEDGEMENT

The PURLs Surveillance System was supported in part by Grant Number UL1RR024999 from the National Center for Research Resources, a Clinical Translational Science Award to the University of Chicago. The content is solely the responsibility of the authors and does not necessarily represent the official views of the National Center for Research Resources or the National Institutes of Health.

Copyright © 2017. The Family Physicians Inquiries Network. All rights reserved.

Reprinted with permission from the Family Physicians Inquiries Network and The Journal of Family Practice (2017;66[3]:174-176).

References

1. Lindson-Hawley N, Banting M, West R, et al. Gradual versus abrupt smoking cessation: a randomized, controlled noninferiority trial. Ann Intern Med. 2016;164:585-592.
2. Hu SS, Neff L, Agaku IT, et al. Tobacco product use among adults—United States, 2013-2014. MMWR Morb Mortal Wkly Rep. 2016;65:685-691.
3. Lavinghouze SR, Malarcher A, Jama A, et al. Trends in quit attempts among adult cigarette smokers–United States, 2001-2013. MMWR Morb Mortal Wkly Rep. 2015;64:1129-1135.
4. Patnode CD, Henderson JT, Thompson JH, et al. Behavioral counseling and pharmacotherapy interventions for tobacco cessation in adults, including pregnant women: a review of reviews for the US Preventive Services Task Force. Ann Intern Med. 2015;163:608-621.
5. Siu AL; US Preventive Services Task Force. Behavioral and pharmacotherapy interventions for tobacco smoking cessation in adults, including pregnant women: US Preventive Services Task Force Recommendation Statement. Ann Intern Med. 2015;163:622-634.
6. Lindson-Hawley N, Aveyard P, Hughes JR. Reduction versus abrupt cessation in smokers who want to quit. Cochrane Database Syst Rev. 2012;11:CD008033.
7. Baha M, Le Faou AL. Gradual versus abrupt quitting among French treatment-seeking smokers. Prev Med. 2014;63: 96-102.
8. West R, Hajek P, Stead L, et al. Outcome criteria in smoking cessation trials: proposal for a common standard. Addiction. 2005;100:299-303.

Article PDF
Author and Disclosure Information

LCDR Dustin K. Smith practices at the Naval Hospital in Jacksonville, Florida. Deborah E. Miller is with the University of Chicago NorthShore Family Medicine Residency in Glenview, Illinois. Anne Mounsey is with the Department of Family Medicine at the University of North Carolina, Chapel Hill.

Disclosures: The views expressed in this article are those of the author(s) and do not necessarily reflect the official policy or position of the Department of the Navy, Department of Defense, or the United States government. Dr. Smith is a military service member. This work was prepared as part of his official duties.

Issue
Clinician Reviews - 27(4)
Publications
Topics
Page Number
26,31-32
Sections
Author and Disclosure Information

LCDR Dustin K. Smith practices at the Naval Hospital in Jacksonville, Florida. Deborah E. Miller is with the University of Chicago NorthShore Family Medicine Residency in Glenview, Illinois. Anne Mounsey is with the Department of Family Medicine at the University of North Carolina, Chapel Hill.

Disclosures: The views expressed in this article are those of the author(s) and do not necessarily reflect the official policy or position of the Department of the Navy, Department of Defense, or the United States government. Dr. Smith is a military service member. This work was prepared as part of his official duties.

Author and Disclosure Information

LCDR Dustin K. Smith practices at the Naval Hospital in Jacksonville, Florida. Deborah E. Miller is with the University of Chicago NorthShore Family Medicine Residency in Glenview, Illinois. Anne Mounsey is with the Department of Family Medicine at the University of North Carolina, Chapel Hill.

Disclosures: The views expressed in this article are those of the author(s) and do not necessarily reflect the official policy or position of the Department of the Navy, Department of Defense, or the United States government. Dr. Smith is a military service member. This work was prepared as part of his official duties.

Article PDF
Article PDF
Related Articles

 

A 43-year-old man has a 35–pack-year smoking history and currently smokes one pack of cigarettes a day. He is eager to quit smoking since a close friend of his was recently diagnosed with lung cancer. He asks whether he should quit “cold turkey” or gradually. What do you recommend?

Between 2013 and 2014, one in five American adults reported using tobacco products some days or every day, and 66% of smokers in 2013 made at least one attempt to quit.2,3 The risks of tobacco use and the benefits of cessation are well established, and behavioral and pharmacologic interventions (both alone and in combination) increase smoking cessation rates.4 The US Preventive Services Task Force recommends that health care providers address tobacco use and cessation with patients at regular office visits and offer behavioral and pharmacologic interventions.5 Current guidelines, however, make no specific recommendations regarding gradual versus abrupt smoking cessation methods.5

A previous Cochrane review of 10 RCTs demonstrated no significant difference in quit rates between gradual cigarette reduction and abrupt cessation. The meta-analysis was limited, however, by differences in patient populations, outcome definitions, and types of interventions (both pharmacologic and behavioral).6

In a retrospective cohort study, French investigators reviewed an online database of more than 60,000 smokers who presented to nationwide cessation services. The researchers found that older participants (those 45 and older) and heavy smokers (≥ 21 cigarettes/d) were more likely to quit gradually than abruptly.7

STUDY SUMMARY

“Cold turkey” is better than gradual cessation at six months

A noninferiority RCT was conducted in England to assess whether gradual smoking cessation is as successful as abrupt cessation.1 The primary outcome was abstinence from smoking at four weeks, assessed using the Russell Standard. This set of six criteria (including validation by exhaled CO concentrations of < 10 ppm) is used by the National Centre for Smoking Cessation and Training to decrease variability of reported smoking cessation rates in English studies.8

Participants were recruited via letters from their primary care practice inviting them to participate in a smoking cessation study. The 697 subjects were randomized to either the abrupt-cessation group or the gradual-cessation group. Baseline characteristics were similar between groups.

All participants were asked to schedule a quit date for two weeks after their enrollment. Patients assigned to the gradual-cessation group were provided nicotine replacement patches (21 mg/d) and their choice of short-acting nicotine replacement therapy (NRT; gum, lozenges, nasal spray, sublingual tablets, inhalator, or mouth spray) to use in the two weeks leading up to the quit date. They were given instructions to reduce smoking by half of the baseline amount by the end of the first week, and to a quarter of baseline by the end of the second week.

Patients randomly assigned to the abrupt-cessation group were instructed to continue their current smoking habits until the cessation date; during those two weeks they were given nicotine patches (because the other group received them, and some evidence suggests that precessation NRT increases quit rates) but no short-acting NRT.

Following the cessation date, treatment in both groups was identical, including behavioral support, nicotine patches (21 mg/d), and the patient’s choice of short-acting NRT. Behavioral support consisted of visits with a research nurse at the patient’s primary care practice at the following intervals: weekly for two weeks before the quit date; the day before the quit date; weekly for four weeks after the quit date; and eight weeks after the quit date.

The chosen noninferiority margin was equal to a relative risk (RR) of 0.81 (19% reduction in effectiveness) of quitting gradually, compared with abrupt cessation of smoking. Quit rates in the gradual-reduction group did not reach the threshold for noninferiority; in fact, four-week abstinence was significantly more likely in the abrupt-cessation group than in the gradual-cessation group (49% vs 39.2%; RR, 0.80; number needed to treat [NNT], 10). Similarly, secondary outcomes of eight-week and six-month abstinence rates showed superiority of abrupt over gradual cessation. Six months after the quit date, 15.5% of the gradual-cessation group and 22% of the abrupt-cessation group remained abstinent (RR, 0.71; NNT, 15).

 

 

 

Patient preference plays a role

The investigators also found a difference in successful cessation based on the participants’ preferred method of cessation. Participants who preferred abrupt cessation were more likely to be abstinent at four weeks than participants who preferred gradual cessation (52.2% vs 38.3%).

Patients with a baseline preference for gradual cessation were equally as likely to successfully quit when allocated to abrupt cessation against their preference as when they were allocated to gradual cessation. Four-week abstinence was seen in 34.6% of patients who preferred and were allocated to gradual cessation and in 42% of patients who preferred gradual but were allocated to abrupt cessation.

WHAT’S NEW

Higher quality study; added element of preference

This large, well-designed, noninferiority study showed that abrupt cessation is superior to gradual cessation. The size and design of the study, including a standardized method of assessing cessation and a standardized intervention, make this a higher quality study than those in the Cochrane meta-analysis.6 This study also showed that participants who preferred gradual cessation were less likely to be successful—regardless of the method to which they were assigned.

CAVEATS

Generalizability limited by race and number of cigarettes smoked

Patients lost to follow-up at four weeks (35 in the abrupt-cessation group and 48 in the gradual-cessation group) were assumed to have continued smoking, which may have biased the results toward abrupt cessation. That said, the large number of study participants, along with the relatively small number lost to follow-up, minimizes this weakness.

The majority of participants were white, which may limit generalizability to nonwhite populations. In addition, participants smoked an average of 20 cigarettes per day and, as noted previously, an observational study of tobacco users in France found that heavy smokers (≥ 21 cigarettes/d) were more likely to quit gradually than abruptly. Therefore, results may not be generalizable to heavy smokers.7

CHALLENGES TO IMPLEMENTATION

Considerable investment in behavioral support

One significant challenge is the implementation of such a structured tobacco cessation program in primary care. Both abrupt- and gradual-cessation groups were given considerable behavioral support from research nurses. Participants in this study were seen by a nurse seven times in the first six weeks of the study, and the intervention included nurse-created reduction schedules.

Even if patients in the study preferred one method of cessation to another, they were receptive to quitting either gradually or abruptly. In clinical practice, patients are often set in their desired method of cessation. In that setting, our role is then to inform them of the data and support them in whatever method they choose.

ACKNOWLEDGEMENT

The PURLs Surveillance System was supported in part by Grant Number UL1RR024999 from the National Center for Research Resources, a Clinical Translational Science Award to the University of Chicago. The content is solely the responsibility of the authors and does not necessarily represent the official views of the National Center for Research Resources or the National Institutes of Health.

Copyright © 2017. The Family Physicians Inquiries Network. All rights reserved.

Reprinted with permission from the Family Physicians Inquiries Network and The Journal of Family Practice (2017;66[3]:174-176).

 

A 43-year-old man has a 35–pack-year smoking history and currently smokes one pack of cigarettes a day. He is eager to quit smoking since a close friend of his was recently diagnosed with lung cancer. He asks whether he should quit “cold turkey” or gradually. What do you recommend?

Between 2013 and 2014, one in five American adults reported using tobacco products some days or every day, and 66% of smokers in 2013 made at least one attempt to quit.2,3 The risks of tobacco use and the benefits of cessation are well established, and behavioral and pharmacologic interventions (both alone and in combination) increase smoking cessation rates.4 The US Preventive Services Task Force recommends that health care providers address tobacco use and cessation with patients at regular office visits and offer behavioral and pharmacologic interventions.5 Current guidelines, however, make no specific recommendations regarding gradual versus abrupt smoking cessation methods.5

A previous Cochrane review of 10 RCTs demonstrated no significant difference in quit rates between gradual cigarette reduction and abrupt cessation. The meta-analysis was limited, however, by differences in patient populations, outcome definitions, and types of interventions (both pharmacologic and behavioral).6

In a retrospective cohort study, French investigators reviewed an online database of more than 60,000 smokers who presented to nationwide cessation services. The researchers found that older participants (those 45 and older) and heavy smokers (≥ 21 cigarettes/d) were more likely to quit gradually than abruptly.7

STUDY SUMMARY

“Cold turkey” is better than gradual cessation at six months

A noninferiority RCT was conducted in England to assess whether gradual smoking cessation is as successful as abrupt cessation.1 The primary outcome was abstinence from smoking at four weeks, assessed using the Russell Standard. This set of six criteria (including validation by exhaled CO concentrations of < 10 ppm) is used by the National Centre for Smoking Cessation and Training to decrease variability of reported smoking cessation rates in English studies.8

Participants were recruited via letters from their primary care practice inviting them to participate in a smoking cessation study. The 697 subjects were randomized to either the abrupt-cessation group or the gradual-cessation group. Baseline characteristics were similar between groups.

All participants were asked to schedule a quit date for two weeks after their enrollment. Patients assigned to the gradual-cessation group were provided nicotine replacement patches (21 mg/d) and their choice of short-acting nicotine replacement therapy (NRT; gum, lozenges, nasal spray, sublingual tablets, inhalator, or mouth spray) to use in the two weeks leading up to the quit date. They were given instructions to reduce smoking by half of the baseline amount by the end of the first week, and to a quarter of baseline by the end of the second week.

Patients randomly assigned to the abrupt-cessation group were instructed to continue their current smoking habits until the cessation date; during those two weeks they were given nicotine patches (because the other group received them, and some evidence suggests that precessation NRT increases quit rates) but no short-acting NRT.

Following the cessation date, treatment in both groups was identical, including behavioral support, nicotine patches (21 mg/d), and the patient’s choice of short-acting NRT. Behavioral support consisted of visits with a research nurse at the patient’s primary care practice at the following intervals: weekly for two weeks before the quit date; the day before the quit date; weekly for four weeks after the quit date; and eight weeks after the quit date.

The chosen noninferiority margin was equal to a relative risk (RR) of 0.81 (19% reduction in effectiveness) of quitting gradually, compared with abrupt cessation of smoking. Quit rates in the gradual-reduction group did not reach the threshold for noninferiority; in fact, four-week abstinence was significantly more likely in the abrupt-cessation group than in the gradual-cessation group (49% vs 39.2%; RR, 0.80; number needed to treat [NNT], 10). Similarly, secondary outcomes of eight-week and six-month abstinence rates showed superiority of abrupt over gradual cessation. Six months after the quit date, 15.5% of the gradual-cessation group and 22% of the abrupt-cessation group remained abstinent (RR, 0.71; NNT, 15).

 

 

 

Patient preference plays a role

The investigators also found a difference in successful cessation based on the participants’ preferred method of cessation. Participants who preferred abrupt cessation were more likely to be abstinent at four weeks than participants who preferred gradual cessation (52.2% vs 38.3%).

Patients with a baseline preference for gradual cessation were equally as likely to successfully quit when allocated to abrupt cessation against their preference as when they were allocated to gradual cessation. Four-week abstinence was seen in 34.6% of patients who preferred and were allocated to gradual cessation and in 42% of patients who preferred gradual but were allocated to abrupt cessation.

WHAT’S NEW

Higher quality study; added element of preference

This large, well-designed, noninferiority study showed that abrupt cessation is superior to gradual cessation. The size and design of the study, including a standardized method of assessing cessation and a standardized intervention, make this a higher quality study than those in the Cochrane meta-analysis.6 This study also showed that participants who preferred gradual cessation were less likely to be successful—regardless of the method to which they were assigned.

CAVEATS

Generalizability limited by race and number of cigarettes smoked

Patients lost to follow-up at four weeks (35 in the abrupt-cessation group and 48 in the gradual-cessation group) were assumed to have continued smoking, which may have biased the results toward abrupt cessation. That said, the large number of study participants, along with the relatively small number lost to follow-up, minimizes this weakness.

The majority of participants were white, which may limit generalizability to nonwhite populations. In addition, participants smoked an average of 20 cigarettes per day and, as noted previously, an observational study of tobacco users in France found that heavy smokers (≥ 21 cigarettes/d) were more likely to quit gradually than abruptly. Therefore, results may not be generalizable to heavy smokers.7

CHALLENGES TO IMPLEMENTATION

Considerable investment in behavioral support

One significant challenge is the implementation of such a structured tobacco cessation program in primary care. Both abrupt- and gradual-cessation groups were given considerable behavioral support from research nurses. Participants in this study were seen by a nurse seven times in the first six weeks of the study, and the intervention included nurse-created reduction schedules.

Even if patients in the study preferred one method of cessation to another, they were receptive to quitting either gradually or abruptly. In clinical practice, patients are often set in their desired method of cessation. In that setting, our role is then to inform them of the data and support them in whatever method they choose.

ACKNOWLEDGEMENT

The PURLs Surveillance System was supported in part by Grant Number UL1RR024999 from the National Center for Research Resources, a Clinical Translational Science Award to the University of Chicago. The content is solely the responsibility of the authors and does not necessarily represent the official views of the National Center for Research Resources or the National Institutes of Health.

Copyright © 2017. The Family Physicians Inquiries Network. All rights reserved.

Reprinted with permission from the Family Physicians Inquiries Network and The Journal of Family Practice (2017;66[3]:174-176).

References

1. Lindson-Hawley N, Banting M, West R, et al. Gradual versus abrupt smoking cessation: a randomized, controlled noninferiority trial. Ann Intern Med. 2016;164:585-592.
2. Hu SS, Neff L, Agaku IT, et al. Tobacco product use among adults—United States, 2013-2014. MMWR Morb Mortal Wkly Rep. 2016;65:685-691.
3. Lavinghouze SR, Malarcher A, Jama A, et al. Trends in quit attempts among adult cigarette smokers–United States, 2001-2013. MMWR Morb Mortal Wkly Rep. 2015;64:1129-1135.
4. Patnode CD, Henderson JT, Thompson JH, et al. Behavioral counseling and pharmacotherapy interventions for tobacco cessation in adults, including pregnant women: a review of reviews for the US Preventive Services Task Force. Ann Intern Med. 2015;163:608-621.
5. Siu AL; US Preventive Services Task Force. Behavioral and pharmacotherapy interventions for tobacco smoking cessation in adults, including pregnant women: US Preventive Services Task Force Recommendation Statement. Ann Intern Med. 2015;163:622-634.
6. Lindson-Hawley N, Aveyard P, Hughes JR. Reduction versus abrupt cessation in smokers who want to quit. Cochrane Database Syst Rev. 2012;11:CD008033.
7. Baha M, Le Faou AL. Gradual versus abrupt quitting among French treatment-seeking smokers. Prev Med. 2014;63: 96-102.
8. West R, Hajek P, Stead L, et al. Outcome criteria in smoking cessation trials: proposal for a common standard. Addiction. 2005;100:299-303.

References

1. Lindson-Hawley N, Banting M, West R, et al. Gradual versus abrupt smoking cessation: a randomized, controlled noninferiority trial. Ann Intern Med. 2016;164:585-592.
2. Hu SS, Neff L, Agaku IT, et al. Tobacco product use among adults—United States, 2013-2014. MMWR Morb Mortal Wkly Rep. 2016;65:685-691.
3. Lavinghouze SR, Malarcher A, Jama A, et al. Trends in quit attempts among adult cigarette smokers–United States, 2001-2013. MMWR Morb Mortal Wkly Rep. 2015;64:1129-1135.
4. Patnode CD, Henderson JT, Thompson JH, et al. Behavioral counseling and pharmacotherapy interventions for tobacco cessation in adults, including pregnant women: a review of reviews for the US Preventive Services Task Force. Ann Intern Med. 2015;163:608-621.
5. Siu AL; US Preventive Services Task Force. Behavioral and pharmacotherapy interventions for tobacco smoking cessation in adults, including pregnant women: US Preventive Services Task Force Recommendation Statement. Ann Intern Med. 2015;163:622-634.
6. Lindson-Hawley N, Aveyard P, Hughes JR. Reduction versus abrupt cessation in smokers who want to quit. Cochrane Database Syst Rev. 2012;11:CD008033.
7. Baha M, Le Faou AL. Gradual versus abrupt quitting among French treatment-seeking smokers. Prev Med. 2014;63: 96-102.
8. West R, Hajek P, Stead L, et al. Outcome criteria in smoking cessation trials: proposal for a common standard. Addiction. 2005;100:299-303.

Issue
Clinician Reviews - 27(4)
Issue
Clinician Reviews - 27(4)
Page Number
26,31-32
Page Number
26,31-32
Publications
Publications
Topics
Article Type
Display Headline
“Cold Turkey” Works Best for Smoking Cessation
Display Headline
“Cold Turkey” Works Best for Smoking Cessation
Sections
Disallow All Ads
Content Gating
No Gating (article Unlocked/Free)
Alternative CME
Article PDF Media

When can exercise supplant surgery for degenerative meniscal tears?

Article Type
Changed
Tue, 06/23/2020 - 13:37
Display Headline
When can exercise supplant surgery for degenerative meniscal tears?
 

ILLUSTRATIVE CASE

A 48-year-old man presents to your office for follow-up of right knee pain that has been bothering him for the last 12 months. He denies any trauma or inciting incident for the pain. On physical exam, he does not have crepitus, but has medial joint line tenderness of his right knee. A magnetic resonance image (MRI) shows a partial, medial meniscal tear. Do you refer him to Physical Therapy (PT) or Orthopedics for arthroscopy and repair?

The meniscus—cartilage in the knee joint that provides support, stability, and lubrication to the joint during activity—can tear during a traumatic event or because of degeneration over time. Traumatic meniscal tears typically happen to younger adults and teens (<30 years of age) during sports, such as basketball and soccer,whereas degenerative meniscal tears generally present in patients ages 40 to 60 years.2,3 The annual incidence of all meniscal tears is 79 per 100,000.4 While some physicians can diagnose traumatic meniscal tears based on history and physical examination, degenerative meniscal tears are generally more challenging, and typically warrant an MRI for confirmation.3

Meniscal tears can be treated either conservatively, with supportive care and exercise, or with surgery. Unfortunately, there are no national orthopedic guidelines available to help direct care. In one observational study of surgery as treatment for both traumatic and degenerative meniscal tears, 95 out of 117 patients (81.2%) were generally satisfied with this treatment at the 4-year follow-up, with higher satisfaction in the traumatic meniscal tear group than in the degenerative tear group.5

Two systematic reviews of surgery vs nonoperative management or sham therapies found no additional benefit of surgery for meniscal tears in a variety of patients with and without osteoarthritis.6,7 However, both studies were of only moderate quality because of the number of patients in the nonoperative groups who ultimately obtained surgery. And neither of the studies directly compared surgery to nonoperative management.6,7

Yet another investigation, a multicenter, randomized, double-blind, sham-controlled study conducted in Finland involving 146 patients, compared sham surgery to arthroscopic partial meniscectomy. Both groups received instruction on performing post-procedure exercises, and both groups had similar and marked improvement in pain and function.8

Clinical practice recommendations devised from a systematic and vast review of the literature recommend that the decision for surgery be based on patient-specific factors such as symptoms, age, mechanism of tear, extent of damage, and occupational/­social/activity needs.9

STUDY SUMMARY

Exercise is as good as—and in one way, better than—surgery

The current randomized controlled superiority trial compared exercise therapy to arthroscopic partial meniscectomy in patients ages 35 to 60 years presenting to the orthopedic departments of 2 hospitals in Norway with unilateral knee pain for more than 2 months and an MRI-delineated medial meniscal tear. Patients were included only if they had radiographic evidence of minimal osteoarthritis (Kellgren-Lawrence classification grade ≤2). Exclusion criteria were acute trauma, locked knee, ligament injury, and knee surgery in the same knee within the previous 2 years.

The primary outcomes were change in patient-reported knee function as determined by overall knee injury and osteoarthritis outcome score (KOOS4) after 2 years and thigh muscle strength at 3 months as measured by physiotherapists. The KOOS4 consists of 4 out of the 5 KOOS subscales: pain, other symptoms (swelling, grinding/noise from the joint, ability to straighten and bend), function in sports/recreation, and knee-related quality of life (QOL). This study utilized the average score of each subscale.

Secondary outcomes were the 5 individual KOOS subscales (the 4 previously mentioned plus activities of daily living [ADLs]), as well as thigh muscle strength and lower extremity performance test results.

Methods. Testing personnel were blinded to group allocation; participants wore pants or neoprene sleeves to cover surgical scars. A total of 140 patients were randomized to either 12 weeks (24-36 sessions) of exercise therapy alone or a standardized arthroscopic partial meniscectomy with written and oral encouragement upon discharge to perform simple exercises at home 2 to4 times daily (to regain range of motion and reduce swelling).

Results. The overall mean improvement in KOOS4 score from baseline at 2 years was similar between the exercise group and the meniscectomy group (25.3 points vs 24.4 points, respectively; mean difference [MD], 0.9; 95% confidence interval [CI], -4.3 to 6.1; P=.72). Additionally, muscle strength (measured as peak torque flexion and extension and total work flexion and extension) at both 3 and 12 months showed significant objective improvements favoring exercise therapy.

Exercise therapy was as effective as surgery after a 2-year follow-up period and was superior in the short term for thigh muscle strength.

Secondary outcomes comparing the change from baseline of KOOS subscale scores showed 4 of the 5 having non-significant differences (pain, ADL, sports/­recreation, and QOL). Only the symptoms subscale had a significant difference favoring exercise therapy (MD, 5.3 points; 95% CI, 0.5 to 10.2; P=.03), which was likely clinically insignificant when using a grading scale of 0 to 100.

Of those patients allocated to exercise therapy alone, 19% crossed over and underwent surgery during the 2 years of the study.

 

 

 

WHAT'S NEW

Head-to-head comparison adds evidence to previous findings

This is the first trial to directly compare exercise therapy to surgery in patients with meniscal tears. Interestingly, exercise therapy was as effective after a 2-year follow-up period and was superior in the short term for thigh muscle strength.1 The results of this study build on those from the smaller study conducted in Finland mentioned earlier.8 In that study, both groups received instruction for the same graduated exercise plan. The researchers found that exercise was comparable to surgery for meniscal tears in patients with no osteoarthritis.

CAVEATS

Results may not translate to those with more severe osteoarthritis

This trial included patients with only mild to no osteoarthritis in addition to their meniscal tear.1 It is unclear if the results would be maintained in patients with more advanced disease. Additionally, 19% of patients crossed over from the exercise group to the surgery group, even though muscle strength improved. Therefore, education about the risks of surgery and the potential lack of benefit is important.

CHALLENGES TO IMPLEMENTATION

The cost and effort of physical therapy may be a deterrent

The cost of PT can be a barrier for some patients who have adequate insurance coverage for surgery, but inadequate coverage for PT. Additionally, exercise therapy requires significant and ongoing amounts of time and effort, which may be a deterrent for patients with busy lifestyles. Patients and physicians may view surgery as an “easier” fix.

ACKNOWLEDGEMENT

The PURLs Surveillance System was supported in part by Grant Number UL1RR024999 from the National Center For Research Resources, a Clinical Translational Science Award to the University of Chicago. The content is solely the responsibility of the authors and does not necessarily represent the official views of the National Center For Research Resources or the National Institutes of Health.

Files
References

1. Kise NJ, Risberg MA, Stensrud S, et al. Exercise therapy versus arthroscopic partial meniscectomy for degenerative meniscal tear in middle aged patients: randomised controlled trial with two year follow-up. BMJ. 2016;354:i3740.

2. Beals CT, Magnussen RA, Graham WC, et al. The prevalence of meniscal pathology in asymptomatic athletes. Sports Med. 2016;46:1517-1524.

3. Maffulli N, Longo UG, Campi S, et al. Meniscal tears. Open Access J Sports Med. 2010;1:45-54.

4. Peat G, Bergknut C, Frobell R, et al. Population-wide incidence estimates for soft tissue knee injuries presenting to healthcare in southern Sweden: data from the Skåne Healthcare Register. Arthritis Res Ther. 2014;16:R162.

5. Ghislain NA, Wei JN, Li YG. Study of the clinical outcome between traumatic and degenerative (non-traumatic) meniscal tears after arthroscopic surgery: a 4-years follow-up study. J Clin Diagn Res. 2016;10:RC01-RC04.

6. Khan M, Evaniew N, Bedi A, et al. Arthroscopic surgery for degenerative tears of the meniscus: a systematic review and meta-analysis. CMAJ. 2014;186:1057-1064.

7. Monk P, Garfjeld Roberts P, Palmer AJR, et al. The urgent need for evidence in arthroscopic meniscal surgery: a systematic review of the evidence for operative management of meniscal tears. Am J Sports Med. 2016;pii: 0363546516650180. [Epub ahead of print]

8. Sihvonen R, Paavola M, Malmivaara A, et al; Finnish Degenerative Meniscal Lesion Study (FIDELITY) Group. Arthroscopic partial meniscectomy versus sham surgery for a degenerative meniscal tear. N Engl J Med. 2013;369:2515-2524.

9. Beaufils P, Hulet C, Dhénain M, et al. Clinical practice guidelines for the management of meniscal lesions and isolated lesions of the anterior cruciate ligament of the knee in adults. Orthop Traumatol Surg Res. 2009;95:437-442.

Article PDF
Author and Disclosure Information

University of Illinois at Chicago (Dr. Jarrett); University of Pittsburgh Medical Center St. Margaret Family Medicine Residency Program, Pa (Dr. Sauereisen)

DEPUTY EDITOR
Corey Lyon, DO

University of Colorado Family Medicine Residency, Denver

Issue
The Journal of Family Practice - 66(4)
Publications
Topics
Page Number
250-252
Sections
Files
Files
Author and Disclosure Information

University of Illinois at Chicago (Dr. Jarrett); University of Pittsburgh Medical Center St. Margaret Family Medicine Residency Program, Pa (Dr. Sauereisen)

DEPUTY EDITOR
Corey Lyon, DO

University of Colorado Family Medicine Residency, Denver

Author and Disclosure Information

University of Illinois at Chicago (Dr. Jarrett); University of Pittsburgh Medical Center St. Margaret Family Medicine Residency Program, Pa (Dr. Sauereisen)

DEPUTY EDITOR
Corey Lyon, DO

University of Colorado Family Medicine Residency, Denver

Article PDF
Article PDF
 

ILLUSTRATIVE CASE

A 48-year-old man presents to your office for follow-up of right knee pain that has been bothering him for the last 12 months. He denies any trauma or inciting incident for the pain. On physical exam, he does not have crepitus, but has medial joint line tenderness of his right knee. A magnetic resonance image (MRI) shows a partial, medial meniscal tear. Do you refer him to Physical Therapy (PT) or Orthopedics for arthroscopy and repair?

The meniscus—cartilage in the knee joint that provides support, stability, and lubrication to the joint during activity—can tear during a traumatic event or because of degeneration over time. Traumatic meniscal tears typically happen to younger adults and teens (<30 years of age) during sports, such as basketball and soccer,whereas degenerative meniscal tears generally present in patients ages 40 to 60 years.2,3 The annual incidence of all meniscal tears is 79 per 100,000.4 While some physicians can diagnose traumatic meniscal tears based on history and physical examination, degenerative meniscal tears are generally more challenging, and typically warrant an MRI for confirmation.3

Meniscal tears can be treated either conservatively, with supportive care and exercise, or with surgery. Unfortunately, there are no national orthopedic guidelines available to help direct care. In one observational study of surgery as treatment for both traumatic and degenerative meniscal tears, 95 out of 117 patients (81.2%) were generally satisfied with this treatment at the 4-year follow-up, with higher satisfaction in the traumatic meniscal tear group than in the degenerative tear group.5

Two systematic reviews of surgery vs nonoperative management or sham therapies found no additional benefit of surgery for meniscal tears in a variety of patients with and without osteoarthritis.6,7 However, both studies were of only moderate quality because of the number of patients in the nonoperative groups who ultimately obtained surgery. And neither of the studies directly compared surgery to nonoperative management.6,7

Yet another investigation, a multicenter, randomized, double-blind, sham-controlled study conducted in Finland involving 146 patients, compared sham surgery to arthroscopic partial meniscectomy. Both groups received instruction on performing post-procedure exercises, and both groups had similar and marked improvement in pain and function.8

Clinical practice recommendations devised from a systematic and vast review of the literature recommend that the decision for surgery be based on patient-specific factors such as symptoms, age, mechanism of tear, extent of damage, and occupational/­social/activity needs.9

STUDY SUMMARY

Exercise is as good as—and in one way, better than—surgery

The current randomized controlled superiority trial compared exercise therapy to arthroscopic partial meniscectomy in patients ages 35 to 60 years presenting to the orthopedic departments of 2 hospitals in Norway with unilateral knee pain for more than 2 months and an MRI-delineated medial meniscal tear. Patients were included only if they had radiographic evidence of minimal osteoarthritis (Kellgren-Lawrence classification grade ≤2). Exclusion criteria were acute trauma, locked knee, ligament injury, and knee surgery in the same knee within the previous 2 years.

The primary outcomes were change in patient-reported knee function as determined by overall knee injury and osteoarthritis outcome score (KOOS4) after 2 years and thigh muscle strength at 3 months as measured by physiotherapists. The KOOS4 consists of 4 out of the 5 KOOS subscales: pain, other symptoms (swelling, grinding/noise from the joint, ability to straighten and bend), function in sports/recreation, and knee-related quality of life (QOL). This study utilized the average score of each subscale.

Secondary outcomes were the 5 individual KOOS subscales (the 4 previously mentioned plus activities of daily living [ADLs]), as well as thigh muscle strength and lower extremity performance test results.

Methods. Testing personnel were blinded to group allocation; participants wore pants or neoprene sleeves to cover surgical scars. A total of 140 patients were randomized to either 12 weeks (24-36 sessions) of exercise therapy alone or a standardized arthroscopic partial meniscectomy with written and oral encouragement upon discharge to perform simple exercises at home 2 to4 times daily (to regain range of motion and reduce swelling).

Results. The overall mean improvement in KOOS4 score from baseline at 2 years was similar between the exercise group and the meniscectomy group (25.3 points vs 24.4 points, respectively; mean difference [MD], 0.9; 95% confidence interval [CI], -4.3 to 6.1; P=.72). Additionally, muscle strength (measured as peak torque flexion and extension and total work flexion and extension) at both 3 and 12 months showed significant objective improvements favoring exercise therapy.

Exercise therapy was as effective as surgery after a 2-year follow-up period and was superior in the short term for thigh muscle strength.

Secondary outcomes comparing the change from baseline of KOOS subscale scores showed 4 of the 5 having non-significant differences (pain, ADL, sports/­recreation, and QOL). Only the symptoms subscale had a significant difference favoring exercise therapy (MD, 5.3 points; 95% CI, 0.5 to 10.2; P=.03), which was likely clinically insignificant when using a grading scale of 0 to 100.

Of those patients allocated to exercise therapy alone, 19% crossed over and underwent surgery during the 2 years of the study.

 

 

 

WHAT'S NEW

Head-to-head comparison adds evidence to previous findings

This is the first trial to directly compare exercise therapy to surgery in patients with meniscal tears. Interestingly, exercise therapy was as effective after a 2-year follow-up period and was superior in the short term for thigh muscle strength.1 The results of this study build on those from the smaller study conducted in Finland mentioned earlier.8 In that study, both groups received instruction for the same graduated exercise plan. The researchers found that exercise was comparable to surgery for meniscal tears in patients with no osteoarthritis.

CAVEATS

Results may not translate to those with more severe osteoarthritis

This trial included patients with only mild to no osteoarthritis in addition to their meniscal tear.1 It is unclear if the results would be maintained in patients with more advanced disease. Additionally, 19% of patients crossed over from the exercise group to the surgery group, even though muscle strength improved. Therefore, education about the risks of surgery and the potential lack of benefit is important.

CHALLENGES TO IMPLEMENTATION

The cost and effort of physical therapy may be a deterrent

The cost of PT can be a barrier for some patients who have adequate insurance coverage for surgery, but inadequate coverage for PT. Additionally, exercise therapy requires significant and ongoing amounts of time and effort, which may be a deterrent for patients with busy lifestyles. Patients and physicians may view surgery as an “easier” fix.

ACKNOWLEDGEMENT

The PURLs Surveillance System was supported in part by Grant Number UL1RR024999 from the National Center For Research Resources, a Clinical Translational Science Award to the University of Chicago. The content is solely the responsibility of the authors and does not necessarily represent the official views of the National Center For Research Resources or the National Institutes of Health.

 

ILLUSTRATIVE CASE

A 48-year-old man presents to your office for follow-up of right knee pain that has been bothering him for the last 12 months. He denies any trauma or inciting incident for the pain. On physical exam, he does not have crepitus, but has medial joint line tenderness of his right knee. A magnetic resonance image (MRI) shows a partial, medial meniscal tear. Do you refer him to Physical Therapy (PT) or Orthopedics for arthroscopy and repair?

The meniscus—cartilage in the knee joint that provides support, stability, and lubrication to the joint during activity—can tear during a traumatic event or because of degeneration over time. Traumatic meniscal tears typically happen to younger adults and teens (<30 years of age) during sports, such as basketball and soccer,whereas degenerative meniscal tears generally present in patients ages 40 to 60 years.2,3 The annual incidence of all meniscal tears is 79 per 100,000.4 While some physicians can diagnose traumatic meniscal tears based on history and physical examination, degenerative meniscal tears are generally more challenging, and typically warrant an MRI for confirmation.3

Meniscal tears can be treated either conservatively, with supportive care and exercise, or with surgery. Unfortunately, there are no national orthopedic guidelines available to help direct care. In one observational study of surgery as treatment for both traumatic and degenerative meniscal tears, 95 out of 117 patients (81.2%) were generally satisfied with this treatment at the 4-year follow-up, with higher satisfaction in the traumatic meniscal tear group than in the degenerative tear group.5

Two systematic reviews of surgery vs nonoperative management or sham therapies found no additional benefit of surgery for meniscal tears in a variety of patients with and without osteoarthritis.6,7 However, both studies were of only moderate quality because of the number of patients in the nonoperative groups who ultimately obtained surgery. And neither of the studies directly compared surgery to nonoperative management.6,7

Yet another investigation, a multicenter, randomized, double-blind, sham-controlled study conducted in Finland involving 146 patients, compared sham surgery to arthroscopic partial meniscectomy. Both groups received instruction on performing post-procedure exercises, and both groups had similar and marked improvement in pain and function.8

Clinical practice recommendations devised from a systematic and vast review of the literature recommend that the decision for surgery be based on patient-specific factors such as symptoms, age, mechanism of tear, extent of damage, and occupational/­social/activity needs.9

STUDY SUMMARY

Exercise is as good as—and in one way, better than—surgery

The current randomized controlled superiority trial compared exercise therapy to arthroscopic partial meniscectomy in patients ages 35 to 60 years presenting to the orthopedic departments of 2 hospitals in Norway with unilateral knee pain for more than 2 months and an MRI-delineated medial meniscal tear. Patients were included only if they had radiographic evidence of minimal osteoarthritis (Kellgren-Lawrence classification grade ≤2). Exclusion criteria were acute trauma, locked knee, ligament injury, and knee surgery in the same knee within the previous 2 years.

The primary outcomes were change in patient-reported knee function as determined by overall knee injury and osteoarthritis outcome score (KOOS4) after 2 years and thigh muscle strength at 3 months as measured by physiotherapists. The KOOS4 consists of 4 out of the 5 KOOS subscales: pain, other symptoms (swelling, grinding/noise from the joint, ability to straighten and bend), function in sports/recreation, and knee-related quality of life (QOL). This study utilized the average score of each subscale.

Secondary outcomes were the 5 individual KOOS subscales (the 4 previously mentioned plus activities of daily living [ADLs]), as well as thigh muscle strength and lower extremity performance test results.

Methods. Testing personnel were blinded to group allocation; participants wore pants or neoprene sleeves to cover surgical scars. A total of 140 patients were randomized to either 12 weeks (24-36 sessions) of exercise therapy alone or a standardized arthroscopic partial meniscectomy with written and oral encouragement upon discharge to perform simple exercises at home 2 to4 times daily (to regain range of motion and reduce swelling).

Results. The overall mean improvement in KOOS4 score from baseline at 2 years was similar between the exercise group and the meniscectomy group (25.3 points vs 24.4 points, respectively; mean difference [MD], 0.9; 95% confidence interval [CI], -4.3 to 6.1; P=.72). Additionally, muscle strength (measured as peak torque flexion and extension and total work flexion and extension) at both 3 and 12 months showed significant objective improvements favoring exercise therapy.

Exercise therapy was as effective as surgery after a 2-year follow-up period and was superior in the short term for thigh muscle strength.

Secondary outcomes comparing the change from baseline of KOOS subscale scores showed 4 of the 5 having non-significant differences (pain, ADL, sports/­recreation, and QOL). Only the symptoms subscale had a significant difference favoring exercise therapy (MD, 5.3 points; 95% CI, 0.5 to 10.2; P=.03), which was likely clinically insignificant when using a grading scale of 0 to 100.

Of those patients allocated to exercise therapy alone, 19% crossed over and underwent surgery during the 2 years of the study.

 

 

 

WHAT'S NEW

Head-to-head comparison adds evidence to previous findings

This is the first trial to directly compare exercise therapy to surgery in patients with meniscal tears. Interestingly, exercise therapy was as effective after a 2-year follow-up period and was superior in the short term for thigh muscle strength.1 The results of this study build on those from the smaller study conducted in Finland mentioned earlier.8 In that study, both groups received instruction for the same graduated exercise plan. The researchers found that exercise was comparable to surgery for meniscal tears in patients with no osteoarthritis.

CAVEATS

Results may not translate to those with more severe osteoarthritis

This trial included patients with only mild to no osteoarthritis in addition to their meniscal tear.1 It is unclear if the results would be maintained in patients with more advanced disease. Additionally, 19% of patients crossed over from the exercise group to the surgery group, even though muscle strength improved. Therefore, education about the risks of surgery and the potential lack of benefit is important.

CHALLENGES TO IMPLEMENTATION

The cost and effort of physical therapy may be a deterrent

The cost of PT can be a barrier for some patients who have adequate insurance coverage for surgery, but inadequate coverage for PT. Additionally, exercise therapy requires significant and ongoing amounts of time and effort, which may be a deterrent for patients with busy lifestyles. Patients and physicians may view surgery as an “easier” fix.

ACKNOWLEDGEMENT

The PURLs Surveillance System was supported in part by Grant Number UL1RR024999 from the National Center For Research Resources, a Clinical Translational Science Award to the University of Chicago. The content is solely the responsibility of the authors and does not necessarily represent the official views of the National Center For Research Resources or the National Institutes of Health.

References

1. Kise NJ, Risberg MA, Stensrud S, et al. Exercise therapy versus arthroscopic partial meniscectomy for degenerative meniscal tear in middle aged patients: randomised controlled trial with two year follow-up. BMJ. 2016;354:i3740.

2. Beals CT, Magnussen RA, Graham WC, et al. The prevalence of meniscal pathology in asymptomatic athletes. Sports Med. 2016;46:1517-1524.

3. Maffulli N, Longo UG, Campi S, et al. Meniscal tears. Open Access J Sports Med. 2010;1:45-54.

4. Peat G, Bergknut C, Frobell R, et al. Population-wide incidence estimates for soft tissue knee injuries presenting to healthcare in southern Sweden: data from the Skåne Healthcare Register. Arthritis Res Ther. 2014;16:R162.

5. Ghislain NA, Wei JN, Li YG. Study of the clinical outcome between traumatic and degenerative (non-traumatic) meniscal tears after arthroscopic surgery: a 4-years follow-up study. J Clin Diagn Res. 2016;10:RC01-RC04.

6. Khan M, Evaniew N, Bedi A, et al. Arthroscopic surgery for degenerative tears of the meniscus: a systematic review and meta-analysis. CMAJ. 2014;186:1057-1064.

7. Monk P, Garfjeld Roberts P, Palmer AJR, et al. The urgent need for evidence in arthroscopic meniscal surgery: a systematic review of the evidence for operative management of meniscal tears. Am J Sports Med. 2016;pii: 0363546516650180. [Epub ahead of print]

8. Sihvonen R, Paavola M, Malmivaara A, et al; Finnish Degenerative Meniscal Lesion Study (FIDELITY) Group. Arthroscopic partial meniscectomy versus sham surgery for a degenerative meniscal tear. N Engl J Med. 2013;369:2515-2524.

9. Beaufils P, Hulet C, Dhénain M, et al. Clinical practice guidelines for the management of meniscal lesions and isolated lesions of the anterior cruciate ligament of the knee in adults. Orthop Traumatol Surg Res. 2009;95:437-442.

References

1. Kise NJ, Risberg MA, Stensrud S, et al. Exercise therapy versus arthroscopic partial meniscectomy for degenerative meniscal tear in middle aged patients: randomised controlled trial with two year follow-up. BMJ. 2016;354:i3740.

2. Beals CT, Magnussen RA, Graham WC, et al. The prevalence of meniscal pathology in asymptomatic athletes. Sports Med. 2016;46:1517-1524.

3. Maffulli N, Longo UG, Campi S, et al. Meniscal tears. Open Access J Sports Med. 2010;1:45-54.

4. Peat G, Bergknut C, Frobell R, et al. Population-wide incidence estimates for soft tissue knee injuries presenting to healthcare in southern Sweden: data from the Skåne Healthcare Register. Arthritis Res Ther. 2014;16:R162.

5. Ghislain NA, Wei JN, Li YG. Study of the clinical outcome between traumatic and degenerative (non-traumatic) meniscal tears after arthroscopic surgery: a 4-years follow-up study. J Clin Diagn Res. 2016;10:RC01-RC04.

6. Khan M, Evaniew N, Bedi A, et al. Arthroscopic surgery for degenerative tears of the meniscus: a systematic review and meta-analysis. CMAJ. 2014;186:1057-1064.

7. Monk P, Garfjeld Roberts P, Palmer AJR, et al. The urgent need for evidence in arthroscopic meniscal surgery: a systematic review of the evidence for operative management of meniscal tears. Am J Sports Med. 2016;pii: 0363546516650180. [Epub ahead of print]

8. Sihvonen R, Paavola M, Malmivaara A, et al; Finnish Degenerative Meniscal Lesion Study (FIDELITY) Group. Arthroscopic partial meniscectomy versus sham surgery for a degenerative meniscal tear. N Engl J Med. 2013;369:2515-2524.

9. Beaufils P, Hulet C, Dhénain M, et al. Clinical practice guidelines for the management of meniscal lesions and isolated lesions of the anterior cruciate ligament of the knee in adults. Orthop Traumatol Surg Res. 2009;95:437-442.

Issue
The Journal of Family Practice - 66(4)
Issue
The Journal of Family Practice - 66(4)
Page Number
250-252
Page Number
250-252
Publications
Publications
Topics
Article Type
Display Headline
When can exercise supplant surgery for degenerative meniscal tears?
Display Headline
When can exercise supplant surgery for degenerative meniscal tears?
Sections
PURLs Copyright

Copyright © 2017. The Family Physicians Inquiries Network. All rights reserved.

Inside the Article

PRACTICE CHANGER

Recommend supervised exercise therapy to your patients with a medial, degenerative meniscal tear and a minimal history of osteoarthritis because it is as effective as partial meniscectomy, entails little risk, and has the added benefit of increasing muscle strength.1

STRENGTH OF RECOMMENDATION

B: Based on a single, good quality, randomized controlled trial.

Kise NJ, Risberg MA, Stensrud S, et al. Exercise therapy versus arthroscopic partial meniscectomy for degenerative meniscal tear in middle aged patients: randomised controlled trial with two year follow-up. BMJ. 2016;354:i3740.

Disallow All Ads
Content Gating
No Gating (article Unlocked/Free)
Alternative CME
PubMed ID
28375398
Use ProPublica
Hide sidebar & use full width
render the right sidebar.
Conference Recap Checkbox
Not Conference Recap
Clinical Edge
Article PDF Media
Media Files

Steroids During Late Preterm Labor? Better Later Than Never

Article Type
Changed
Wed, 03/15/2017 - 02:31
Display Headline
Steroids During Late Preterm Labor? Better Later Than Never

 

A 21-year-old G1P0 at 35 weeks, 2 days of gestation presents to labor and delivery reporting a “gush of clear fluid.” On exam, you confirm she has preterm rupture of membranes. She is contracting every three minutes and has a cervix dilated to 3 cm. Is there any neonatal benefit to using corticosteroids in this late preterm period?

Approximately 12% of all births in the United States are the result of preterm labor, and 8% take place in the late preterm period, defined as 34 to 36 weeks’ gestation.2,3 To reduce risk for neonatal death and respiratory complications, both the American College of Obstetricians and Gynecologists and the National Institutes of Health recommend a course of corticosteroids between 24 and 34 weeks’ gestation for women at increased risk for preterm delivery.2,4 Due to a lack of evidence from RCTs on the benefit of corticosteroids in late preterm labor, there are no recommendations to extend this period.5 However, multiple studies have shown that babies born during the late preterm period have more neonatal complications than term newborns.6-8

A retrospective chart review of more than 130,000 live births found that newborns who were delivered between 34 and 36 weeks had higher rates of respiratory distress than those delivered at 39 weeks (ventilator use dropped from 3.3% at 34 weeks to 0.3% at 39 weeks, and transient tachypnea decreased from 2.4% at 34 weeks to 0.4% at 39 weeks).6 Another retrospective review of more than 230,000 newborns, 19,000 of whom were born in the late preterm period, revealed that more neonates born between 34 and 36 weeks’ gestation had respiratory distress syndrome than neonates delivered at 39 weeks (10.5% at 34 weeks, 6% at 35 weeks, 2.8% at 36 weeks vs 0.3% at 39 weeks).8

STUDY SUMMARY

Late preterm newborns breathe better with antenatal betamethasone

This RCT examined the effectiveness of betamethasone in preventing neonatal respiratory complications for 2,831 women at high probability of preterm delivery between 34 weeks and 36 weeks, 6 days of gestation. “High probability of preterm delivery” was defined as preterm labor with intact membranes and at least 3 cm dilation or 75% cervical effacement; spontaneous rupture of membranes; or anticipated preterm delivery for any other indication either through induction or cesarean section between 24 hours and seven days after the planned randomization.

Patients were randomly assigned to receive two intramuscular injections (12 mg each) of either betamethasone or placebo, 24 hours apart. The two doses were successfully given in 60% of the betamethasone group and 59% of the placebo group. In 95% of the cases in which the second dose was not given, it was because delivery occurred within 24 hours of the first dose.

The primary outcome was the need for respiratory support within 72 hours of birth, defined as one or more of the following: the use of continuous positive airway pressure (CPAP) or high-flow nasal cannula for at least two consecutive hours, supplemental oxygen for at least four continuous hours, extracorporeal membrane oxygenation (ECMO), or mechanical ventilation.

The median length of time from enrollment to delivery was 31 to 33 hours, and 31.4% underwent cesarean delivery. In the intention-to-treat analysis, the primary outcome was significantly lower in the betamethasone group than in the placebo group (11.6% vs 14.4%; relative risk [RR], 0.80; number needed to treat [NNT], 35). Secondary outcomes (severe complications, representing a composite of the use of CPAP or high-flow nasal cannula for at least 12 continuous hours, supplemental oxygen for at least 24 continuous hours, ECMO, mechanical ventilation, stillbirth, or neonatal death within 72 hours after delivery) were also lower in the betamethasone group (8.1% vs 12.1%; RR, 0.67; NNT, 25). The betamethasone group also had a lower risk for transient tachypnea of the newborn (6.7% vs 9.9%; RR, 0.68).

There were no significant differences in the occurrence of maternal chorioamnio­nitis or endometritis between the groups. Hypoglycemia in the newborn occurred more in the betamethasone group (24% vs 15%; RR, 1.6; number needed to harm [NNH], 11). The betamethasone group had two neonatal deaths: one from septic shock, and the other from a structural cardiac anomaly and arrhythmia.

WHAT’S NEW

Betamethasone effective even in the late, late preterm period

This study demonstrated an improvement in neonatal respiratory outcomes when betamethasone versus placebo was used in the late preterm period. The findings were similar to those from the Antenatal Steroids for Term Elective Caesarean Section Research Team.9 Their trial showed a reduction in respiratory complications in term neonates delivered via elective cesarean section to mothers who received antenatal betamethasone (NNT, 37, to prevent admission to a special care nursery with respiratory distress). The findings were also consistent with those of a recent meta-analysis evaluating the occurrence of respiratory complications with the use of antenatal betamethasone in women expected to deliver in the late preterm period or with a planned cesarean delivery at ≥ 37 weeks’ gestation.10

 

 

 

CAVEATS

Neonates may develop hypoglycemia

The authors of the study reported an increased risk for hypoglycemia in the neonates receiving antenatal betamethasone. The long-term implications of this are unclear, however, given that there was a reduction in intermediate care nursery and neonatal ICU stays that were three days or longer in the betamethasone group. There was also no difference in hospital length of stay between the two groups. Additionally, it’s unclear if there are any long-term neonatal complications of betamethasone use in the late preterm period.

CHALLENGES TO IMPLEMENTATION

Challenges are negligible

There are minimal challenges to implementing this strategy, as betamethasone is routinely used for preterm labor and is readily available on labor and delivery units.

ACKNOWLEDGEMENT

The PURLs Surveillance System was supported in part by Grant Number UL1RR024999 from the National Center For Research Resources, a Clinical Translational Science Award to the University of Chicago. The content is solely the responsibility of the authors and does not necessarily represent the official views of the National Center For Research Resources or the National Institutes of Health.

Copyright © 2017. The Family Physicians Inquiries Network. All rights reserved.

Reprinted with permission from the Family Physicians Inquiries Network and The Journal of Family Practice. 2017;66(2):104-106.

References

1. Gyamfi-Bannerman C, Thom EA, Blackwell SC, et al; NICHD Maternal–Fetal Medicine Units Network. Antenatal betamethasone for women at risk for late preterm delivery. N Engl J Med. 2016;374:1311-1320.
2. Practice Bulletin No. 159 Summary: Management of Preterm Labor. Obstet Gynecol. 2016;127:190-191.
3. Martin JA, Hamilton BE, Osterman MJ, et al. Births: final data for 2013. Natl Vital Stat Rep. 2015;64:1-65.
4. Effect of corticosteroids for fetal maturation on perinatal outcomes. NIH Consens Statement. 1994;12:1-24.
5. Society for Maternal-Fetal Medicine (SMFM) Publications Committee. Implementation of the use of antenatal corticosteroids in the late preterm birth period in women at risk for preterm delivery. Am J Obstet Gynecol. 2016;215:B13-B15.
6. McIntire DD, Leveno KJ. Neonatal mortality and morbidity rates in late preterm births compared with births at term. Obstet Gynecol. 2008;111:35-41.
7. Yoder BA, Gordon MC, Barth WH Jr. Late-preterm birth: does the changing obstetric paradigm alter the epidemiology of respiratory complications? Obstet Gynecol. 2008;111:814-822.
8. Consortium on Safe Labor, Hibbard JU, Wilkins I, Sun L, et al. Respiratory morbidity in late preterm births. JAMA. 2010;304:419-425.
9. Stutchfield P, Whitaker R, Russell I. Antenatal betamethasone and incidence of neonatal respiratory distress after elective caesarean section: pragmatic randomised trial. BMJ. 2005;331:662.
10. Saccone G, Berghella V. Antenatal corticosteroids for maturity of term or near term fetuses: systematic review and meta-analysis of randomized controlled trials. BMJ. 2016;355:i5044.

Article PDF
Author and Disclosure Information

Corey Lyon is with the University of Colorado Family Medicine Residency, Denver. Jennifer K. Bello is with the St. Louis University School of Medicine, Missouri.

Issue
Clinician Reviews - 27(3)
Publications
Topics
Page Number
37-39
Sections
Author and Disclosure Information

Corey Lyon is with the University of Colorado Family Medicine Residency, Denver. Jennifer K. Bello is with the St. Louis University School of Medicine, Missouri.

Author and Disclosure Information

Corey Lyon is with the University of Colorado Family Medicine Residency, Denver. Jennifer K. Bello is with the St. Louis University School of Medicine, Missouri.

Article PDF
Article PDF
Related Articles

 

A 21-year-old G1P0 at 35 weeks, 2 days of gestation presents to labor and delivery reporting a “gush of clear fluid.” On exam, you confirm she has preterm rupture of membranes. She is contracting every three minutes and has a cervix dilated to 3 cm. Is there any neonatal benefit to using corticosteroids in this late preterm period?

Approximately 12% of all births in the United States are the result of preterm labor, and 8% take place in the late preterm period, defined as 34 to 36 weeks’ gestation.2,3 To reduce risk for neonatal death and respiratory complications, both the American College of Obstetricians and Gynecologists and the National Institutes of Health recommend a course of corticosteroids between 24 and 34 weeks’ gestation for women at increased risk for preterm delivery.2,4 Due to a lack of evidence from RCTs on the benefit of corticosteroids in late preterm labor, there are no recommendations to extend this period.5 However, multiple studies have shown that babies born during the late preterm period have more neonatal complications than term newborns.6-8

A retrospective chart review of more than 130,000 live births found that newborns who were delivered between 34 and 36 weeks had higher rates of respiratory distress than those delivered at 39 weeks (ventilator use dropped from 3.3% at 34 weeks to 0.3% at 39 weeks, and transient tachypnea decreased from 2.4% at 34 weeks to 0.4% at 39 weeks).6 Another retrospective review of more than 230,000 newborns, 19,000 of whom were born in the late preterm period, revealed that more neonates born between 34 and 36 weeks’ gestation had respiratory distress syndrome than neonates delivered at 39 weeks (10.5% at 34 weeks, 6% at 35 weeks, 2.8% at 36 weeks vs 0.3% at 39 weeks).8

STUDY SUMMARY

Late preterm newborns breathe better with antenatal betamethasone

This RCT examined the effectiveness of betamethasone in preventing neonatal respiratory complications for 2,831 women at high probability of preterm delivery between 34 weeks and 36 weeks, 6 days of gestation. “High probability of preterm delivery” was defined as preterm labor with intact membranes and at least 3 cm dilation or 75% cervical effacement; spontaneous rupture of membranes; or anticipated preterm delivery for any other indication either through induction or cesarean section between 24 hours and seven days after the planned randomization.

Patients were randomly assigned to receive two intramuscular injections (12 mg each) of either betamethasone or placebo, 24 hours apart. The two doses were successfully given in 60% of the betamethasone group and 59% of the placebo group. In 95% of the cases in which the second dose was not given, it was because delivery occurred within 24 hours of the first dose.

The primary outcome was the need for respiratory support within 72 hours of birth, defined as one or more of the following: the use of continuous positive airway pressure (CPAP) or high-flow nasal cannula for at least two consecutive hours, supplemental oxygen for at least four continuous hours, extracorporeal membrane oxygenation (ECMO), or mechanical ventilation.

The median length of time from enrollment to delivery was 31 to 33 hours, and 31.4% underwent cesarean delivery. In the intention-to-treat analysis, the primary outcome was significantly lower in the betamethasone group than in the placebo group (11.6% vs 14.4%; relative risk [RR], 0.80; number needed to treat [NNT], 35). Secondary outcomes (severe complications, representing a composite of the use of CPAP or high-flow nasal cannula for at least 12 continuous hours, supplemental oxygen for at least 24 continuous hours, ECMO, mechanical ventilation, stillbirth, or neonatal death within 72 hours after delivery) were also lower in the betamethasone group (8.1% vs 12.1%; RR, 0.67; NNT, 25). The betamethasone group also had a lower risk for transient tachypnea of the newborn (6.7% vs 9.9%; RR, 0.68).

There were no significant differences in the occurrence of maternal chorioamnio­nitis or endometritis between the groups. Hypoglycemia in the newborn occurred more in the betamethasone group (24% vs 15%; RR, 1.6; number needed to harm [NNH], 11). The betamethasone group had two neonatal deaths: one from septic shock, and the other from a structural cardiac anomaly and arrhythmia.

WHAT’S NEW

Betamethasone effective even in the late, late preterm period

This study demonstrated an improvement in neonatal respiratory outcomes when betamethasone versus placebo was used in the late preterm period. The findings were similar to those from the Antenatal Steroids for Term Elective Caesarean Section Research Team.9 Their trial showed a reduction in respiratory complications in term neonates delivered via elective cesarean section to mothers who received antenatal betamethasone (NNT, 37, to prevent admission to a special care nursery with respiratory distress). The findings were also consistent with those of a recent meta-analysis evaluating the occurrence of respiratory complications with the use of antenatal betamethasone in women expected to deliver in the late preterm period or with a planned cesarean delivery at ≥ 37 weeks’ gestation.10

 

 

 

CAVEATS

Neonates may develop hypoglycemia

The authors of the study reported an increased risk for hypoglycemia in the neonates receiving antenatal betamethasone. The long-term implications of this are unclear, however, given that there was a reduction in intermediate care nursery and neonatal ICU stays that were three days or longer in the betamethasone group. There was also no difference in hospital length of stay between the two groups. Additionally, it’s unclear if there are any long-term neonatal complications of betamethasone use in the late preterm period.

CHALLENGES TO IMPLEMENTATION

Challenges are negligible

There are minimal challenges to implementing this strategy, as betamethasone is routinely used for preterm labor and is readily available on labor and delivery units.

ACKNOWLEDGEMENT

The PURLs Surveillance System was supported in part by Grant Number UL1RR024999 from the National Center For Research Resources, a Clinical Translational Science Award to the University of Chicago. The content is solely the responsibility of the authors and does not necessarily represent the official views of the National Center For Research Resources or the National Institutes of Health.

Copyright © 2017. The Family Physicians Inquiries Network. All rights reserved.

Reprinted with permission from the Family Physicians Inquiries Network and The Journal of Family Practice. 2017;66(2):104-106.

 

A 21-year-old G1P0 at 35 weeks, 2 days of gestation presents to labor and delivery reporting a “gush of clear fluid.” On exam, you confirm she has preterm rupture of membranes. She is contracting every three minutes and has a cervix dilated to 3 cm. Is there any neonatal benefit to using corticosteroids in this late preterm period?

Approximately 12% of all births in the United States are the result of preterm labor, and 8% take place in the late preterm period, defined as 34 to 36 weeks’ gestation.2,3 To reduce risk for neonatal death and respiratory complications, both the American College of Obstetricians and Gynecologists and the National Institutes of Health recommend a course of corticosteroids between 24 and 34 weeks’ gestation for women at increased risk for preterm delivery.2,4 Due to a lack of evidence from RCTs on the benefit of corticosteroids in late preterm labor, there are no recommendations to extend this period.5 However, multiple studies have shown that babies born during the late preterm period have more neonatal complications than term newborns.6-8

A retrospective chart review of more than 130,000 live births found that newborns who were delivered between 34 and 36 weeks had higher rates of respiratory distress than those delivered at 39 weeks (ventilator use dropped from 3.3% at 34 weeks to 0.3% at 39 weeks, and transient tachypnea decreased from 2.4% at 34 weeks to 0.4% at 39 weeks).6 Another retrospective review of more than 230,000 newborns, 19,000 of whom were born in the late preterm period, revealed that more neonates born between 34 and 36 weeks’ gestation had respiratory distress syndrome than neonates delivered at 39 weeks (10.5% at 34 weeks, 6% at 35 weeks, 2.8% at 36 weeks vs 0.3% at 39 weeks).8

STUDY SUMMARY

Late preterm newborns breathe better with antenatal betamethasone

This RCT examined the effectiveness of betamethasone in preventing neonatal respiratory complications for 2,831 women at high probability of preterm delivery between 34 weeks and 36 weeks, 6 days of gestation. “High probability of preterm delivery” was defined as preterm labor with intact membranes and at least 3 cm dilation or 75% cervical effacement; spontaneous rupture of membranes; or anticipated preterm delivery for any other indication either through induction or cesarean section between 24 hours and seven days after the planned randomization.

Patients were randomly assigned to receive two intramuscular injections (12 mg each) of either betamethasone or placebo, 24 hours apart. The two doses were successfully given in 60% of the betamethasone group and 59% of the placebo group. In 95% of the cases in which the second dose was not given, it was because delivery occurred within 24 hours of the first dose.

The primary outcome was the need for respiratory support within 72 hours of birth, defined as one or more of the following: the use of continuous positive airway pressure (CPAP) or high-flow nasal cannula for at least two consecutive hours, supplemental oxygen for at least four continuous hours, extracorporeal membrane oxygenation (ECMO), or mechanical ventilation.

The median length of time from enrollment to delivery was 31 to 33 hours, and 31.4% underwent cesarean delivery. In the intention-to-treat analysis, the primary outcome was significantly lower in the betamethasone group than in the placebo group (11.6% vs 14.4%; relative risk [RR], 0.80; number needed to treat [NNT], 35). Secondary outcomes (severe complications, representing a composite of the use of CPAP or high-flow nasal cannula for at least 12 continuous hours, supplemental oxygen for at least 24 continuous hours, ECMO, mechanical ventilation, stillbirth, or neonatal death within 72 hours after delivery) were also lower in the betamethasone group (8.1% vs 12.1%; RR, 0.67; NNT, 25). The betamethasone group also had a lower risk for transient tachypnea of the newborn (6.7% vs 9.9%; RR, 0.68).

There were no significant differences in the occurrence of maternal chorioamnio­nitis or endometritis between the groups. Hypoglycemia in the newborn occurred more in the betamethasone group (24% vs 15%; RR, 1.6; number needed to harm [NNH], 11). The betamethasone group had two neonatal deaths: one from septic shock, and the other from a structural cardiac anomaly and arrhythmia.

WHAT’S NEW

Betamethasone effective even in the late, late preterm period

This study demonstrated an improvement in neonatal respiratory outcomes when betamethasone versus placebo was used in the late preterm period. The findings were similar to those from the Antenatal Steroids for Term Elective Caesarean Section Research Team.9 Their trial showed a reduction in respiratory complications in term neonates delivered via elective cesarean section to mothers who received antenatal betamethasone (NNT, 37, to prevent admission to a special care nursery with respiratory distress). The findings were also consistent with those of a recent meta-analysis evaluating the occurrence of respiratory complications with the use of antenatal betamethasone in women expected to deliver in the late preterm period or with a planned cesarean delivery at ≥ 37 weeks’ gestation.10

 

 

 

CAVEATS

Neonates may develop hypoglycemia

The authors of the study reported an increased risk for hypoglycemia in the neonates receiving antenatal betamethasone. The long-term implications of this are unclear, however, given that there was a reduction in intermediate care nursery and neonatal ICU stays that were three days or longer in the betamethasone group. There was also no difference in hospital length of stay between the two groups. Additionally, it’s unclear if there are any long-term neonatal complications of betamethasone use in the late preterm period.

CHALLENGES TO IMPLEMENTATION

Challenges are negligible

There are minimal challenges to implementing this strategy, as betamethasone is routinely used for preterm labor and is readily available on labor and delivery units.

ACKNOWLEDGEMENT

The PURLs Surveillance System was supported in part by Grant Number UL1RR024999 from the National Center For Research Resources, a Clinical Translational Science Award to the University of Chicago. The content is solely the responsibility of the authors and does not necessarily represent the official views of the National Center For Research Resources or the National Institutes of Health.

Copyright © 2017. The Family Physicians Inquiries Network. All rights reserved.

Reprinted with permission from the Family Physicians Inquiries Network and The Journal of Family Practice. 2017;66(2):104-106.

References

1. Gyamfi-Bannerman C, Thom EA, Blackwell SC, et al; NICHD Maternal–Fetal Medicine Units Network. Antenatal betamethasone for women at risk for late preterm delivery. N Engl J Med. 2016;374:1311-1320.
2. Practice Bulletin No. 159 Summary: Management of Preterm Labor. Obstet Gynecol. 2016;127:190-191.
3. Martin JA, Hamilton BE, Osterman MJ, et al. Births: final data for 2013. Natl Vital Stat Rep. 2015;64:1-65.
4. Effect of corticosteroids for fetal maturation on perinatal outcomes. NIH Consens Statement. 1994;12:1-24.
5. Society for Maternal-Fetal Medicine (SMFM) Publications Committee. Implementation of the use of antenatal corticosteroids in the late preterm birth period in women at risk for preterm delivery. Am J Obstet Gynecol. 2016;215:B13-B15.
6. McIntire DD, Leveno KJ. Neonatal mortality and morbidity rates in late preterm births compared with births at term. Obstet Gynecol. 2008;111:35-41.
7. Yoder BA, Gordon MC, Barth WH Jr. Late-preterm birth: does the changing obstetric paradigm alter the epidemiology of respiratory complications? Obstet Gynecol. 2008;111:814-822.
8. Consortium on Safe Labor, Hibbard JU, Wilkins I, Sun L, et al. Respiratory morbidity in late preterm births. JAMA. 2010;304:419-425.
9. Stutchfield P, Whitaker R, Russell I. Antenatal betamethasone and incidence of neonatal respiratory distress after elective caesarean section: pragmatic randomised trial. BMJ. 2005;331:662.
10. Saccone G, Berghella V. Antenatal corticosteroids for maturity of term or near term fetuses: systematic review and meta-analysis of randomized controlled trials. BMJ. 2016;355:i5044.

References

1. Gyamfi-Bannerman C, Thom EA, Blackwell SC, et al; NICHD Maternal–Fetal Medicine Units Network. Antenatal betamethasone for women at risk for late preterm delivery. N Engl J Med. 2016;374:1311-1320.
2. Practice Bulletin No. 159 Summary: Management of Preterm Labor. Obstet Gynecol. 2016;127:190-191.
3. Martin JA, Hamilton BE, Osterman MJ, et al. Births: final data for 2013. Natl Vital Stat Rep. 2015;64:1-65.
4. Effect of corticosteroids for fetal maturation on perinatal outcomes. NIH Consens Statement. 1994;12:1-24.
5. Society for Maternal-Fetal Medicine (SMFM) Publications Committee. Implementation of the use of antenatal corticosteroids in the late preterm birth period in women at risk for preterm delivery. Am J Obstet Gynecol. 2016;215:B13-B15.
6. McIntire DD, Leveno KJ. Neonatal mortality and morbidity rates in late preterm births compared with births at term. Obstet Gynecol. 2008;111:35-41.
7. Yoder BA, Gordon MC, Barth WH Jr. Late-preterm birth: does the changing obstetric paradigm alter the epidemiology of respiratory complications? Obstet Gynecol. 2008;111:814-822.
8. Consortium on Safe Labor, Hibbard JU, Wilkins I, Sun L, et al. Respiratory morbidity in late preterm births. JAMA. 2010;304:419-425.
9. Stutchfield P, Whitaker R, Russell I. Antenatal betamethasone and incidence of neonatal respiratory distress after elective caesarean section: pragmatic randomised trial. BMJ. 2005;331:662.
10. Saccone G, Berghella V. Antenatal corticosteroids for maturity of term or near term fetuses: systematic review and meta-analysis of randomized controlled trials. BMJ. 2016;355:i5044.

Issue
Clinician Reviews - 27(3)
Issue
Clinician Reviews - 27(3)
Page Number
37-39
Page Number
37-39
Publications
Publications
Topics
Article Type
Display Headline
Steroids During Late Preterm Labor? Better Later Than Never
Display Headline
Steroids During Late Preterm Labor? Better Later Than Never
Sections
Disallow All Ads
Content Gating
No Gating (article Unlocked/Free)
Alternative CME
Article PDF Media

“Cold turkey” works best for smoking cessation

Article Type
Changed
Tue, 06/23/2020 - 13:40
Display Headline
“Cold turkey” works best for smoking cessation
 

ILLUSTRATIVE CASE

A 43-year-old man has a 35-pack-year smoking history and currently smokes a pack of cigarettes a day. He is eager to quit smoking after recently learning that a close friend of his has been diagnosed with lung cancer. He asks you whether he should quit “cold turkey” or gradually. What would you recommend?

Between 2013 and 2014, one in 5 American adults reported using tobacco products some days or every day, and 66% of smokers in 2013 made at least one attempt to quit.2,3 The risks of tobacco use and the benefits of cessation are well established, and behavioral and pharmacologic interventions both alone and in combination increase smoking cessation rates.4 The US Preventive Services Task Force recommends that health care providers address tobacco use and cessation with patients at regular office visits and offer behavioral and pharmacologic interventions.5 Current guidelines, however, make no specific recommendations regarding gradual vs abrupt smoking cessation methods.5

A previous Cochrane review of 10 randomized controlled trials demonstrated no significant difference in quit rates between gradual cigarette reduction leading up to a designated quit day and abrupt cessation. The meta-analysis was limited, however, by differences in patient populations, outcome definitions, and types of interventions (both pharmacologic and behavioral).6

In a retrospective cohort study, French investigators reviewed an online database of 62,508 smokers who presented to nationwide cessation services. The researchers found that older participants (≥45 years of age) and heavy smokers (≥21 cigarettes/d) were more likely to quit gradually than abruptly.7

STUDY SUMMARY

Quitting “cold turkey” is better than gradual cessation at 6 months

Lindson-Hawley, et al, conducted a randomized, controlled, non-inferiority trial in England to assess if gradual cessation is as successful as abrupt cessation as a means of quitting smoking.1 The primary outcome was abstinence from smoking at 4 weeks, assessed using the Russell Standard, a set of 6 standard criteria (including validation by exhaled carbon monoxide concentrations of <10 ppm) used by the National Centre for Smoking Cessation and Training to decrease variability of reported smoking cessation rates in English studies.8

Study participants were recruited via letters from their primary care practice inviting them to call the researchers if they were interested in participating in a smoking cessation study. Almost 1100 people inquired about the study. In the end, 697 were randomized to either the abrupt-cessation group (n=355) or the gradual-cessation group (n=342). Baseline characteristics between the 2 groups were similar.

All participants were asked to schedule a quit date for 2 weeks after their enrollment. Patients randomized to the gradual-cessation group were provided nicotine replacement patches (21 mg/d) and their choice of short-acting nicotine replacement therapy (NRT) (gum, lozenges, nasal spray, sublingual tablets, inhalator, or mouth spray) to use in the 2 weeks leading up to the quit date, along with instructions to reduce smoking by half of the baseline amount by the end of the first week, and to a quarter of baseline by the end of the second week.

Patients randomized to the abrupt-cessation group were instructed to continue their current smoking habits until the cessation date; during those 2 weeks they were given nicotine patches (because the other group received them and some evidence suggests that precessation NRT increases quit rates), but no short-acting NRT.

Following the cessation date, treatment in both groups was identical, including behavioral support, 21 mg/d nicotine patches, and the participant’s choice of short-acting NRT. Behavioral support consisted of visits with a research nurse at the patient’s primary care practice weekly for 2 weeks before the quit date, the day before the quit date, weekly for 4 weeks after the quit date, and 8 weeks after the quit date.

The chosen non-inferiority margin was equal to a relative risk (RR) of 0.81 (19% reduction in effectiveness) of quitting gradually compared with abrupt cessation of smoking. Quit rates in the gradual-reduction group did not reach the threshold for non-inferiority; in fact, 4-week abstinence was significantly more likely in the abrupt-cessation group (49%) than in the gradual-cessation group (39.2%) (RR=0.80; 95% confidence interval [CI], 0.66-0.93; number needed to treat [NNT]=10). Similarly, secondary outcomes of 8-week and 6-month abstinence rates showed superiority of abrupt over gradual cessation. At 6 months after the quit date, 15.5% of the gradual-cessation group and 22% of the abrupt-cessation group remained abstinent (RR=0.71; 95% CI, 0.46-0.91; NNT=15).

 

 

 

Patients’ preferred method of cessation plays a role

The investigators also found a difference in successful cessation based on the participants preferred method of cessation. Participants who preferred abrupt cessation were more likely to be abstinent at 4 weeks than participants who preferred gradual cessation (52.2% vs 38.3%; P=.007).

People who prefer gradual cessation are less likely to be successful at quitting—regardless of whether they try to quit abruptly or gradually.Patients with a baseline preference for gradual cessation were equally as likely to successfully quit when allocated to abrupt cessation against their preference as when they were allocated to gradual cessation: 4-week abstinence was seen in 34.6% of patients who preferred gradual cessation and were allocated to gradual cessation and in 42% of patients who preferred gradual cessation but were allocated to abrupt cessation (P=.152).

WHAT'S NEW

Higher quality than previous studies and added element of preference

This large, well-designed, non-inferiority study showed that abrupt cessation is superior to gradual cessation. The size and design of the study, including a standardized method of assessing cessation and a standardized intervention, make this a higher quality study than those in the Cochrane meta-analysis.6 This study also showed that participants who preferred gradual cessation were less likely to be successful—regardless of the method to which they were ultimately randomized.

CAVEATS

Generalizability limited by race and number of cigarettes smoked

Patients lost to follow-up at 4 weeks (35 in the abrupt-cessation group and 48 in the gradual-cessation group) were assumed to have continued smoking, which may have biased the results toward abrupt cessation. That said, the large number of participants included in the study, along with the relatively small number of patients lost to follow-up, minimizes this weakness.

The participants were largely white, which may limit generalizability to non-white populations. In addition, participants smoked an average of 20 cigarettes per day and, as noted previously, an observational study of tobacco users in France found that heavy smokers (≥21 cigarettes/d) were more likely to quit gradually than abruptly, so results may not be generalizable to heavy smokers.7

CHALLENGES TO IMPLEMENTATION

Finding the time and staff for considerable behavioral support

One important challenge is the implementation of such a structured tobacco cessation program in primary care. Both abrupt- and gradual-cessation groups were given considerable behavioral support from research nurses. Participants in this study were seen by a nurse 7 times in the first 6 weeks of the study, and the intervention included nurse-created reduction schedules.

Even if patients in the study preferred one method of cessation to another, they were receptive to quitting either gradually or abruptly. In clinical practice, patients are often set in their desired method of cessation. In that setting, our role is then to inform them of the data and support them in whatever method they choose.

ACKNOWLEDGEMENT

The PURLs Surveillance System was supported in part by Grant Number UL1RR024999 from the National Center or Research Resources, a Clinical Translational Science Award to the University of Chicago. The content is solely the responsibility of the authors and does not necessarily represent the official views of the National Center for Research Resources or the National Institutes of Health.

Files
References

1. Lindson-Hawley N, Banting M, West R, et al. Gradual versus abrupt smoking cessation: a randomized, controlled noninferiority trial. Ann Intern Med. 2016;164:585-592.

2. Hu SS, Neff L, Agaku IT, et al. Tobacco product use among adults—United States, 2013-2014. MMWR Morb Mortal Wkly Rep. 2016;65:685-691.

3. Lavinghouze SR, Malarcher A, Jama A, et al. Trends in quit attempts among adult cigarette smokers–United States, 2001-2013. MMWR Morb Mortal Wkly Rep. 2015;64:1129-1135.

4. Patnode CD, Henderson JT, Thompson JH, et al. Behavioral counseling and pharmacotherapy interventions for tobacco cessation in adults, including pregnant women: a review of reviews for the US Preventive Services Task Force. Ann Intern Med. 2015;163:608-621.

5. Siu AL, for the US Preventive Services Task Force. Behavioral and pharmacotherapy interventions for tobacco smoking cessation in adults, including pregnant women: US Preventive Services Task Force Recommendation Statement. Ann Intern Med. 2015;163:622-634.

6. Lindson-Hawley N, Aveyard P, Hughes JR. Reduction versus abrupt cessation in smokers who want to quit. Cochrane Database Syst Rev. 2012;11:CD008033.

7. Baha M, Le Faou AL. Gradual versus abrupt quitting among French treatment-seeking smokers. Preventive Medicine. 2014;63:96-102.

8. West R, Hajek P, Stead L, et al. Outcome criteria in smoking cessation trials: proposal for a common standard. Addiction. 2005;100:299-303.

Article PDF
Author and Disclosure Information

Naval Hospital Jacksonville, Fla (Dr. Smith); University of Chicago NorthShore Family Medicine Residency, Glenview, Ill (Dr. Miller); Department of Family Medicine, University of North Carolina, Chapel Hill (Dr. Mounsey)

DEPUTY EDITOR
Shailendra Prasad, MBBS, MPH

University of Minnesota North Memorial Family Medicine Residency, Minneapolis

The views expressed in this article are those of the author(s) and do not necessarily reflect the official policy or position of the Department of the Navy, Department of Defense, or the United States government.

Dr. Smith is a military service member. This work was prepared as part of his official duties.

Issue
The Journal of Family Practice - 66(3)
Publications
Topics
Page Number
174-176
Sections
Files
Files
Author and Disclosure Information

Naval Hospital Jacksonville, Fla (Dr. Smith); University of Chicago NorthShore Family Medicine Residency, Glenview, Ill (Dr. Miller); Department of Family Medicine, University of North Carolina, Chapel Hill (Dr. Mounsey)

DEPUTY EDITOR
Shailendra Prasad, MBBS, MPH

University of Minnesota North Memorial Family Medicine Residency, Minneapolis

The views expressed in this article are those of the author(s) and do not necessarily reflect the official policy or position of the Department of the Navy, Department of Defense, or the United States government.

Dr. Smith is a military service member. This work was prepared as part of his official duties.

Author and Disclosure Information

Naval Hospital Jacksonville, Fla (Dr. Smith); University of Chicago NorthShore Family Medicine Residency, Glenview, Ill (Dr. Miller); Department of Family Medicine, University of North Carolina, Chapel Hill (Dr. Mounsey)

DEPUTY EDITOR
Shailendra Prasad, MBBS, MPH

University of Minnesota North Memorial Family Medicine Residency, Minneapolis

The views expressed in this article are those of the author(s) and do not necessarily reflect the official policy or position of the Department of the Navy, Department of Defense, or the United States government.

Dr. Smith is a military service member. This work was prepared as part of his official duties.

Article PDF
Article PDF
Related Articles
 

ILLUSTRATIVE CASE

A 43-year-old man has a 35-pack-year smoking history and currently smokes a pack of cigarettes a day. He is eager to quit smoking after recently learning that a close friend of his has been diagnosed with lung cancer. He asks you whether he should quit “cold turkey” or gradually. What would you recommend?

Between 2013 and 2014, one in 5 American adults reported using tobacco products some days or every day, and 66% of smokers in 2013 made at least one attempt to quit.2,3 The risks of tobacco use and the benefits of cessation are well established, and behavioral and pharmacologic interventions both alone and in combination increase smoking cessation rates.4 The US Preventive Services Task Force recommends that health care providers address tobacco use and cessation with patients at regular office visits and offer behavioral and pharmacologic interventions.5 Current guidelines, however, make no specific recommendations regarding gradual vs abrupt smoking cessation methods.5

A previous Cochrane review of 10 randomized controlled trials demonstrated no significant difference in quit rates between gradual cigarette reduction leading up to a designated quit day and abrupt cessation. The meta-analysis was limited, however, by differences in patient populations, outcome definitions, and types of interventions (both pharmacologic and behavioral).6

In a retrospective cohort study, French investigators reviewed an online database of 62,508 smokers who presented to nationwide cessation services. The researchers found that older participants (≥45 years of age) and heavy smokers (≥21 cigarettes/d) were more likely to quit gradually than abruptly.7

STUDY SUMMARY

Quitting “cold turkey” is better than gradual cessation at 6 months

Lindson-Hawley, et al, conducted a randomized, controlled, non-inferiority trial in England to assess if gradual cessation is as successful as abrupt cessation as a means of quitting smoking.1 The primary outcome was abstinence from smoking at 4 weeks, assessed using the Russell Standard, a set of 6 standard criteria (including validation by exhaled carbon monoxide concentrations of <10 ppm) used by the National Centre for Smoking Cessation and Training to decrease variability of reported smoking cessation rates in English studies.8

Study participants were recruited via letters from their primary care practice inviting them to call the researchers if they were interested in participating in a smoking cessation study. Almost 1100 people inquired about the study. In the end, 697 were randomized to either the abrupt-cessation group (n=355) or the gradual-cessation group (n=342). Baseline characteristics between the 2 groups were similar.

All participants were asked to schedule a quit date for 2 weeks after their enrollment. Patients randomized to the gradual-cessation group were provided nicotine replacement patches (21 mg/d) and their choice of short-acting nicotine replacement therapy (NRT) (gum, lozenges, nasal spray, sublingual tablets, inhalator, or mouth spray) to use in the 2 weeks leading up to the quit date, along with instructions to reduce smoking by half of the baseline amount by the end of the first week, and to a quarter of baseline by the end of the second week.

Patients randomized to the abrupt-cessation group were instructed to continue their current smoking habits until the cessation date; during those 2 weeks they were given nicotine patches (because the other group received them and some evidence suggests that precessation NRT increases quit rates), but no short-acting NRT.

Following the cessation date, treatment in both groups was identical, including behavioral support, 21 mg/d nicotine patches, and the participant’s choice of short-acting NRT. Behavioral support consisted of visits with a research nurse at the patient’s primary care practice weekly for 2 weeks before the quit date, the day before the quit date, weekly for 4 weeks after the quit date, and 8 weeks after the quit date.

The chosen non-inferiority margin was equal to a relative risk (RR) of 0.81 (19% reduction in effectiveness) of quitting gradually compared with abrupt cessation of smoking. Quit rates in the gradual-reduction group did not reach the threshold for non-inferiority; in fact, 4-week abstinence was significantly more likely in the abrupt-cessation group (49%) than in the gradual-cessation group (39.2%) (RR=0.80; 95% confidence interval [CI], 0.66-0.93; number needed to treat [NNT]=10). Similarly, secondary outcomes of 8-week and 6-month abstinence rates showed superiority of abrupt over gradual cessation. At 6 months after the quit date, 15.5% of the gradual-cessation group and 22% of the abrupt-cessation group remained abstinent (RR=0.71; 95% CI, 0.46-0.91; NNT=15).

 

 

 

Patients’ preferred method of cessation plays a role

The investigators also found a difference in successful cessation based on the participants preferred method of cessation. Participants who preferred abrupt cessation were more likely to be abstinent at 4 weeks than participants who preferred gradual cessation (52.2% vs 38.3%; P=.007).

People who prefer gradual cessation are less likely to be successful at quitting—regardless of whether they try to quit abruptly or gradually.Patients with a baseline preference for gradual cessation were equally as likely to successfully quit when allocated to abrupt cessation against their preference as when they were allocated to gradual cessation: 4-week abstinence was seen in 34.6% of patients who preferred gradual cessation and were allocated to gradual cessation and in 42% of patients who preferred gradual cessation but were allocated to abrupt cessation (P=.152).

WHAT'S NEW

Higher quality than previous studies and added element of preference

This large, well-designed, non-inferiority study showed that abrupt cessation is superior to gradual cessation. The size and design of the study, including a standardized method of assessing cessation and a standardized intervention, make this a higher quality study than those in the Cochrane meta-analysis.6 This study also showed that participants who preferred gradual cessation were less likely to be successful—regardless of the method to which they were ultimately randomized.

CAVEATS

Generalizability limited by race and number of cigarettes smoked

Patients lost to follow-up at 4 weeks (35 in the abrupt-cessation group and 48 in the gradual-cessation group) were assumed to have continued smoking, which may have biased the results toward abrupt cessation. That said, the large number of participants included in the study, along with the relatively small number of patients lost to follow-up, minimizes this weakness.

The participants were largely white, which may limit generalizability to non-white populations. In addition, participants smoked an average of 20 cigarettes per day and, as noted previously, an observational study of tobacco users in France found that heavy smokers (≥21 cigarettes/d) were more likely to quit gradually than abruptly, so results may not be generalizable to heavy smokers.7

CHALLENGES TO IMPLEMENTATION

Finding the time and staff for considerable behavioral support

One important challenge is the implementation of such a structured tobacco cessation program in primary care. Both abrupt- and gradual-cessation groups were given considerable behavioral support from research nurses. Participants in this study were seen by a nurse 7 times in the first 6 weeks of the study, and the intervention included nurse-created reduction schedules.

Even if patients in the study preferred one method of cessation to another, they were receptive to quitting either gradually or abruptly. In clinical practice, patients are often set in their desired method of cessation. In that setting, our role is then to inform them of the data and support them in whatever method they choose.

ACKNOWLEDGEMENT

The PURLs Surveillance System was supported in part by Grant Number UL1RR024999 from the National Center or Research Resources, a Clinical Translational Science Award to the University of Chicago. The content is solely the responsibility of the authors and does not necessarily represent the official views of the National Center for Research Resources or the National Institutes of Health.

 

ILLUSTRATIVE CASE

A 43-year-old man has a 35-pack-year smoking history and currently smokes a pack of cigarettes a day. He is eager to quit smoking after recently learning that a close friend of his has been diagnosed with lung cancer. He asks you whether he should quit “cold turkey” or gradually. What would you recommend?

Between 2013 and 2014, one in 5 American adults reported using tobacco products some days or every day, and 66% of smokers in 2013 made at least one attempt to quit.2,3 The risks of tobacco use and the benefits of cessation are well established, and behavioral and pharmacologic interventions both alone and in combination increase smoking cessation rates.4 The US Preventive Services Task Force recommends that health care providers address tobacco use and cessation with patients at regular office visits and offer behavioral and pharmacologic interventions.5 Current guidelines, however, make no specific recommendations regarding gradual vs abrupt smoking cessation methods.5

A previous Cochrane review of 10 randomized controlled trials demonstrated no significant difference in quit rates between gradual cigarette reduction leading up to a designated quit day and abrupt cessation. The meta-analysis was limited, however, by differences in patient populations, outcome definitions, and types of interventions (both pharmacologic and behavioral).6

In a retrospective cohort study, French investigators reviewed an online database of 62,508 smokers who presented to nationwide cessation services. The researchers found that older participants (≥45 years of age) and heavy smokers (≥21 cigarettes/d) were more likely to quit gradually than abruptly.7

STUDY SUMMARY

Quitting “cold turkey” is better than gradual cessation at 6 months

Lindson-Hawley, et al, conducted a randomized, controlled, non-inferiority trial in England to assess if gradual cessation is as successful as abrupt cessation as a means of quitting smoking.1 The primary outcome was abstinence from smoking at 4 weeks, assessed using the Russell Standard, a set of 6 standard criteria (including validation by exhaled carbon monoxide concentrations of <10 ppm) used by the National Centre for Smoking Cessation and Training to decrease variability of reported smoking cessation rates in English studies.8

Study participants were recruited via letters from their primary care practice inviting them to call the researchers if they were interested in participating in a smoking cessation study. Almost 1100 people inquired about the study. In the end, 697 were randomized to either the abrupt-cessation group (n=355) or the gradual-cessation group (n=342). Baseline characteristics between the 2 groups were similar.

All participants were asked to schedule a quit date for 2 weeks after their enrollment. Patients randomized to the gradual-cessation group were provided nicotine replacement patches (21 mg/d) and their choice of short-acting nicotine replacement therapy (NRT) (gum, lozenges, nasal spray, sublingual tablets, inhalator, or mouth spray) to use in the 2 weeks leading up to the quit date, along with instructions to reduce smoking by half of the baseline amount by the end of the first week, and to a quarter of baseline by the end of the second week.

Patients randomized to the abrupt-cessation group were instructed to continue their current smoking habits until the cessation date; during those 2 weeks they were given nicotine patches (because the other group received them and some evidence suggests that precessation NRT increases quit rates), but no short-acting NRT.

Following the cessation date, treatment in both groups was identical, including behavioral support, 21 mg/d nicotine patches, and the participant’s choice of short-acting NRT. Behavioral support consisted of visits with a research nurse at the patient’s primary care practice weekly for 2 weeks before the quit date, the day before the quit date, weekly for 4 weeks after the quit date, and 8 weeks after the quit date.

The chosen non-inferiority margin was equal to a relative risk (RR) of 0.81 (19% reduction in effectiveness) of quitting gradually compared with abrupt cessation of smoking. Quit rates in the gradual-reduction group did not reach the threshold for non-inferiority; in fact, 4-week abstinence was significantly more likely in the abrupt-cessation group (49%) than in the gradual-cessation group (39.2%) (RR=0.80; 95% confidence interval [CI], 0.66-0.93; number needed to treat [NNT]=10). Similarly, secondary outcomes of 8-week and 6-month abstinence rates showed superiority of abrupt over gradual cessation. At 6 months after the quit date, 15.5% of the gradual-cessation group and 22% of the abrupt-cessation group remained abstinent (RR=0.71; 95% CI, 0.46-0.91; NNT=15).

 

 

 

Patients’ preferred method of cessation plays a role

The investigators also found a difference in successful cessation based on the participants preferred method of cessation. Participants who preferred abrupt cessation were more likely to be abstinent at 4 weeks than participants who preferred gradual cessation (52.2% vs 38.3%; P=.007).

People who prefer gradual cessation are less likely to be successful at quitting—regardless of whether they try to quit abruptly or gradually.Patients with a baseline preference for gradual cessation were equally as likely to successfully quit when allocated to abrupt cessation against their preference as when they were allocated to gradual cessation: 4-week abstinence was seen in 34.6% of patients who preferred gradual cessation and were allocated to gradual cessation and in 42% of patients who preferred gradual cessation but were allocated to abrupt cessation (P=.152).

WHAT'S NEW

Higher quality than previous studies and added element of preference

This large, well-designed, non-inferiority study showed that abrupt cessation is superior to gradual cessation. The size and design of the study, including a standardized method of assessing cessation and a standardized intervention, make this a higher quality study than those in the Cochrane meta-analysis.6 This study also showed that participants who preferred gradual cessation were less likely to be successful—regardless of the method to which they were ultimately randomized.

CAVEATS

Generalizability limited by race and number of cigarettes smoked

Patients lost to follow-up at 4 weeks (35 in the abrupt-cessation group and 48 in the gradual-cessation group) were assumed to have continued smoking, which may have biased the results toward abrupt cessation. That said, the large number of participants included in the study, along with the relatively small number of patients lost to follow-up, minimizes this weakness.

The participants were largely white, which may limit generalizability to non-white populations. In addition, participants smoked an average of 20 cigarettes per day and, as noted previously, an observational study of tobacco users in France found that heavy smokers (≥21 cigarettes/d) were more likely to quit gradually than abruptly, so results may not be generalizable to heavy smokers.7

CHALLENGES TO IMPLEMENTATION

Finding the time and staff for considerable behavioral support

One important challenge is the implementation of such a structured tobacco cessation program in primary care. Both abrupt- and gradual-cessation groups were given considerable behavioral support from research nurses. Participants in this study were seen by a nurse 7 times in the first 6 weeks of the study, and the intervention included nurse-created reduction schedules.

Even if patients in the study preferred one method of cessation to another, they were receptive to quitting either gradually or abruptly. In clinical practice, patients are often set in their desired method of cessation. In that setting, our role is then to inform them of the data and support them in whatever method they choose.

ACKNOWLEDGEMENT

The PURLs Surveillance System was supported in part by Grant Number UL1RR024999 from the National Center or Research Resources, a Clinical Translational Science Award to the University of Chicago. The content is solely the responsibility of the authors and does not necessarily represent the official views of the National Center for Research Resources or the National Institutes of Health.

References

1. Lindson-Hawley N, Banting M, West R, et al. Gradual versus abrupt smoking cessation: a randomized, controlled noninferiority trial. Ann Intern Med. 2016;164:585-592.

2. Hu SS, Neff L, Agaku IT, et al. Tobacco product use among adults—United States, 2013-2014. MMWR Morb Mortal Wkly Rep. 2016;65:685-691.

3. Lavinghouze SR, Malarcher A, Jama A, et al. Trends in quit attempts among adult cigarette smokers–United States, 2001-2013. MMWR Morb Mortal Wkly Rep. 2015;64:1129-1135.

4. Patnode CD, Henderson JT, Thompson JH, et al. Behavioral counseling and pharmacotherapy interventions for tobacco cessation in adults, including pregnant women: a review of reviews for the US Preventive Services Task Force. Ann Intern Med. 2015;163:608-621.

5. Siu AL, for the US Preventive Services Task Force. Behavioral and pharmacotherapy interventions for tobacco smoking cessation in adults, including pregnant women: US Preventive Services Task Force Recommendation Statement. Ann Intern Med. 2015;163:622-634.

6. Lindson-Hawley N, Aveyard P, Hughes JR. Reduction versus abrupt cessation in smokers who want to quit. Cochrane Database Syst Rev. 2012;11:CD008033.

7. Baha M, Le Faou AL. Gradual versus abrupt quitting among French treatment-seeking smokers. Preventive Medicine. 2014;63:96-102.

8. West R, Hajek P, Stead L, et al. Outcome criteria in smoking cessation trials: proposal for a common standard. Addiction. 2005;100:299-303.

References

1. Lindson-Hawley N, Banting M, West R, et al. Gradual versus abrupt smoking cessation: a randomized, controlled noninferiority trial. Ann Intern Med. 2016;164:585-592.

2. Hu SS, Neff L, Agaku IT, et al. Tobacco product use among adults—United States, 2013-2014. MMWR Morb Mortal Wkly Rep. 2016;65:685-691.

3. Lavinghouze SR, Malarcher A, Jama A, et al. Trends in quit attempts among adult cigarette smokers–United States, 2001-2013. MMWR Morb Mortal Wkly Rep. 2015;64:1129-1135.

4. Patnode CD, Henderson JT, Thompson JH, et al. Behavioral counseling and pharmacotherapy interventions for tobacco cessation in adults, including pregnant women: a review of reviews for the US Preventive Services Task Force. Ann Intern Med. 2015;163:608-621.

5. Siu AL, for the US Preventive Services Task Force. Behavioral and pharmacotherapy interventions for tobacco smoking cessation in adults, including pregnant women: US Preventive Services Task Force Recommendation Statement. Ann Intern Med. 2015;163:622-634.

6. Lindson-Hawley N, Aveyard P, Hughes JR. Reduction versus abrupt cessation in smokers who want to quit. Cochrane Database Syst Rev. 2012;11:CD008033.

7. Baha M, Le Faou AL. Gradual versus abrupt quitting among French treatment-seeking smokers. Preventive Medicine. 2014;63:96-102.

8. West R, Hajek P, Stead L, et al. Outcome criteria in smoking cessation trials: proposal for a common standard. Addiction. 2005;100:299-303.

Issue
The Journal of Family Practice - 66(3)
Issue
The Journal of Family Practice - 66(3)
Page Number
174-176
Page Number
174-176
Publications
Publications
Topics
Article Type
Display Headline
“Cold turkey” works best for smoking cessation
Display Headline
“Cold turkey” works best for smoking cessation
Sections
PURLs Copyright

Copyright © 2017. The Family Physicians Inquiries Network. All rights reserved.

Inside the Article

PRACTICE CHANGER

Counsel patients who want to quit smoking that abrupt smoking cessation is more effective for long-term abstinence than taking a gradual approach.

STRENGTH OF RECOMMENDATION

B: Based on one well-designed, randomized controlled trial.

Lindson-Hawley N, Banting M, West R, et al. Gradual versus abrupt smoking cessation: a randomized, controlled noninferiority trial. Ann Intern Med. 2016;164:585-592.1

Disallow All Ads
Content Gating
No Gating (article Unlocked/Free)
Alternative CME
PubMed ID
28249056
Use ProPublica
Hide sidebar & use full width
render the right sidebar.
Conference Recap Checkbox
Not Conference Recap
Clinical Edge
Article PDF Media
Media Files

Need an Add-on to Metformin? Consider This

Article Type
Changed
Tue, 05/03/2022 - 15:31
Display Headline
Need an Add-on to Metformin? Consider This

 

A 58-year-old woman with T2DM and heart failure returns to your office for follow-up. She has been on the maximum dose of metformin alone for the past six months, but her A1C is now 7.8%. She wants to avoid injections. What do you recommend?

There is surprisingly little consensus about what to add to metformin for patients with T2DM who require a second agent to achieve their glycemic goal. Attaining glycemic control earlier in the course of the disease may lead to reduced overall cardiovascular (CV) risk, so the choice of a second drug is an important one.2 While the proven mortality benefit, wide availability, and low cost of metformin make it well-established as initial pharmacotherapy, no second-choice drug has amassed enough evidence of benefit to become the add-on therapy of choice.

The professional societies are of little assistance; dual-therapy recommendations from the American Diabetes Association and the European Association for the Study of Diabetes do not specify a preference.3 Although the American Association of Clinical Endocrinologists/American College of Endocrinology suggest a hierarchy of choices, it is based on expert consensus recommendations.4

A look at the options

Options for add-on therapy include sulfonylureas, thiazolidines, DPP-4 inhibitors, sodium glucose cotransporter 2 inhibitors, glucagon-like peptide 1 (GLP-1) agonists, and insulin. Providers frequently prescribe sulfonylureas after metformin because they are low in cost, have long-term safety data, and are effective at lowering A1C. They work by directly stimulating insulin secretion via pancreatic ß-cells in a glucose-independent manner. But as a 2010 meta-analysis revealed, sulfonylureas carry significant risk for hypoglycemia (relative risk [RR], 4.57) and weight gain (average, 2.06 kg), compared to placebo.5

DPP-4 inhibitors, on the other hand, induce insulin secretion in a glucose-dependent manner through an incretin mechanism. Combined with metformin, they provide glucose control similar to that achieved with the combination of a sulfonylurea and metformin.6 DPP-4 inhibitors were initially found to be associated with fewer CV events and less hypoglycemia than sulfonylureas but were subsequently linked to an increased risk for heart failure–related hospitalization.7

A recent study provides more data on the effects of DPP-4s added to metformin.1

STUDY SUMMARY

DPP-4s as effective, less risky

This observational cohort study compared DPP-4 inhibitors and sulfonylureas when combined with metformin for the treatment of T2DM.1 Outcomes were all-cause mortality, major adverse CV events (defined as hospitalization for ischemic stroke or myocardial infarction [MI]), and hospitalizations for either heart failure or hypoglycemia. The study included data from the National Health Insurance Research Database in Taiwan on more than 70,000 patients (ages 20 and older) with diagnosed T2DM. Individuals adherent to metformin were considered to be enrolled in the cohort on the day they began using either a DPP-4 inhibitor or a sulfonylurea, in addition to metformin.

The researchers collected additional data on socioeconomic factors, urbanization, robustness of the local health care system, Charlson Comorbidity Index, adapted Diabetes Complications Severity Index, and other comorbidities and medications that could affect the outcomes of interest. Participants were then matched by propensity score into 10,089 pairs, each consisting of one DPP-4 inhibitor user and one sulfonylurea user.

After mean follow-up of 2.8 years, the investigators used Cox regression analysis to evaluate the relative hazards of the outcomes. Subgroup analysis stratified by age, sex, Charlson Comorbidity Index, hypertension, chronic kidney disease, hospitalization for heart failure, MI, and cerebrovascular disease yielded results similar to those of the primary analysis for each outcome. Similar results were also obtained when the data were analyzed without propensity-score matching.

The researchers found that users of DPP-4 inhibitors—compared with those who used sulfonylureas—had a lower risk for all-cause mortality (366 vs 488 deaths; hazard ratio [HR], 0.63; number needed to treat [NNT], 117), major cardiac events (209 vs 282 events; HR, 0.68; NNT, 191), ischemic stroke (144 vs 203 strokes; HR, 0.64; NNT, 246), and hypoglycemia (89 vs 170 events; HR, 0.43; NNT, 201). There were no significant differences in the occurrence of MIs (69 vs 88 MIs; HR, 0.75) or the number of hospitalizations for heart failure (100 vs 100 events; HR, 0.78) between the two groups.

WHAT’S NEW

Lower risks for death, CV events, and hypoglycemia

This study found that when added to metformin, DPP-4 inhibitors were associated with lower risks for all-cause mortality, CV events, and hypoglycemia when compared to sulfonylureas. Additionally, DPP-4 inhibitors did not increase the risk for heart failure hospitalization. A recent multicenter observational study of nearly 1.5 million patients on the effects of incretin-based treatments (including DPP-4 inhibitors and GLP-1 agonists) found no increased risk for heart failure hospitalization with DPP-4 inhibitors, compared to other combinations of oral T2DM agents.8

 

 

 

CAVEATS

Did unmeasured confounders play a role?

Unmeasured confounders potentially bias all observational population cohort results. In this particular study, there may have been unmeasured but significant patient factors that providers used to choose diabetes medications. Also, the study did not evaluate diabetes control, although previous studies have shown similar glucose control between sulfonylureas and DPP-4 inhibitors when added to metformin.6

Another caveat is that the results from this study group may not be generalizable to other populations due to physiologic differences. People of Asian ancestry are at risk for T2DM at a lower BMI than people of European ancestry, which could affect the outcomes of interest.9

Furthermore, the study did not evaluate outcomes based on whether patients were taking first-, second-, or third-generation sulfonylureas. Some sulfonylureas (eg, glyburide) carry a higher risk for hypoglycemia, which could bias the results.10

Lastly, the study only provides guidance when choosing between a sulfonylurea and a DPP-4 inhibitor for secondline pharmacotherapy. The GRADE trial, due to be completed in 2023, is comparing sulfonylureas, DPP-4 inhibitors, GLP-1 agonists, and insulin as add-on medications to metformin; it may provide more data on which to base treatment decisions.11

CHALLENGES TO IMPLEMENTATION

DPP-4s are more expensive

Sulfonylureas and DPP-4 inhibitors are both available as generic medications, but the cost of DPP-4 inhibitors remains significantly higher.12 Higher copays and deductibles could affect patient preference. For patients without health insurance, sulfonylureas are available on the discounted drug lists of many major retailers, while DPP-4 inhibitors are not.

ACKNOWLEDGEMENT

The PURLs Surveillance System was supported in part by Grant Number UL1RR024999 from the National Center For Research Resources, a Clinical Translational Science Award to the University of Chicago. The content is solely the responsibility of the authors and does not necessarily represent the official views of the National Center For Research Resources or the National Institutes of Health.

Copyright © 2017. The Family Physicians Inquiries Network. All rights reserved.

Reprinted with permission from the Family Physicians Inquiries Network and The Journal of Family Practice. 2017;66(1):42-44.

References

1. Ou SM, Shih CJ, Chao PW, et al. Effects of clinical outcomes of adding dipeptidyl peptidase-4 inhibitors versus sulfonylureas to metformin therapy in patients with type 2 diabetes mellitus. Ann Intern Med. 2015;163:663-672.
2. Hayward RA, Reaven PD, Wiitala WL, et al. Follow-up of glycemic control and cardiovascular outcomes in type 2 diabetes. N Engl J Med. 2015;372:2197-2206.
3. American Diabetes Association. Standards of Medical Care in Diabetes—2016. Diabetes Care. 2016;39(suppl 1).
4. Garber AJ, Abrahamson MJ, Barzilay JI, et al. Consensus Statement by the American Association of Clinical Endocrinologists and American College of Endocrinology on the Comprehensive Type 2 Diabetes Management Algorithm—2016 Executive Summary. Endocr Pract. 2016;22: 84-113.
5. Phung OJ, Scholle JM, Talwar M, et al. Effect of noninsulin antidiabetic drugs added to metformin therapy on glycemic control, weight gain, and hypoglycemia in type 2 diabetes. JAMA. 2010;303:1410-1418.
6. Gallwitz B, Rosenstock J, Rauch T, et al. 2-year efficacy and safety of linagliptin compared with glimepiride in patients with type 2 diabetes inadequately controlled on metformin: a randomised, double-blind, non-inferiority trial. Lancet. 2012;380:475-483.
7. Scirica BM, Bhatt DL, Braunwald E, et al. Saxagliptin and cardiovascular outcomes in patients with type 2 diabetes mellitus. N Engl J Med. 2013;369:1317-1326.
8. Filion KB, Azoulay L, Platt RW, et al. A multicenter observational study of incretin-based drugs and heart failure. N Engl J Med. 2016;374:1145-1154.
9. Chan JC, Malik V, Jia W, et al. Diabetes in Asia: epidemiology, risk factors, pathophysiology. JAMA. 2009;301:2129-2140.
10. Gangji AS, Cukierman T, Gerstein HC, et al. A systematic review and meta-analysis of hypoglycemia and cardiovascular events: a comparison of glyburide with other secretagogues and with insulin. Diabetes Care. 2007;30:389-394.
11. Nathan DM, Buse JB, Kahn SE, et al. Rationale and design of the glycemia reduction approaches in diabetes: a comparative effectiveness study (GRADE). Diabetes Care. 2013;36:2254-2261.
12. GoodRx. Gliptins. www.goodrx.com/gliptins. Accessed January 4, 2017.

Article PDF
Author and Disclosure Information

David Wyncott is with the St. Joseph Health System Family Medicine Residency, Mishawaka, Indiana. Corey Lyon is with the University of Colorado Family Medicine Residency, Denver. Anne Mounsey is in the Department of Family Medicine at the University of North Carolina, Chapel Hill.

Issue
Clinician Reviews - 27(2)
Publications
Topics
Page Number
20-22
Sections
Author and Disclosure Information

David Wyncott is with the St. Joseph Health System Family Medicine Residency, Mishawaka, Indiana. Corey Lyon is with the University of Colorado Family Medicine Residency, Denver. Anne Mounsey is in the Department of Family Medicine at the University of North Carolina, Chapel Hill.

Author and Disclosure Information

David Wyncott is with the St. Joseph Health System Family Medicine Residency, Mishawaka, Indiana. Corey Lyon is with the University of Colorado Family Medicine Residency, Denver. Anne Mounsey is in the Department of Family Medicine at the University of North Carolina, Chapel Hill.

Article PDF
Article PDF
Related Articles

 

A 58-year-old woman with T2DM and heart failure returns to your office for follow-up. She has been on the maximum dose of metformin alone for the past six months, but her A1C is now 7.8%. She wants to avoid injections. What do you recommend?

There is surprisingly little consensus about what to add to metformin for patients with T2DM who require a second agent to achieve their glycemic goal. Attaining glycemic control earlier in the course of the disease may lead to reduced overall cardiovascular (CV) risk, so the choice of a second drug is an important one.2 While the proven mortality benefit, wide availability, and low cost of metformin make it well-established as initial pharmacotherapy, no second-choice drug has amassed enough evidence of benefit to become the add-on therapy of choice.

The professional societies are of little assistance; dual-therapy recommendations from the American Diabetes Association and the European Association for the Study of Diabetes do not specify a preference.3 Although the American Association of Clinical Endocrinologists/American College of Endocrinology suggest a hierarchy of choices, it is based on expert consensus recommendations.4

A look at the options

Options for add-on therapy include sulfonylureas, thiazolidines, DPP-4 inhibitors, sodium glucose cotransporter 2 inhibitors, glucagon-like peptide 1 (GLP-1) agonists, and insulin. Providers frequently prescribe sulfonylureas after metformin because they are low in cost, have long-term safety data, and are effective at lowering A1C. They work by directly stimulating insulin secretion via pancreatic ß-cells in a glucose-independent manner. But as a 2010 meta-analysis revealed, sulfonylureas carry significant risk for hypoglycemia (relative risk [RR], 4.57) and weight gain (average, 2.06 kg), compared to placebo.5

DPP-4 inhibitors, on the other hand, induce insulin secretion in a glucose-dependent manner through an incretin mechanism. Combined with metformin, they provide glucose control similar to that achieved with the combination of a sulfonylurea and metformin.6 DPP-4 inhibitors were initially found to be associated with fewer CV events and less hypoglycemia than sulfonylureas but were subsequently linked to an increased risk for heart failure–related hospitalization.7

A recent study provides more data on the effects of DPP-4s added to metformin.1

STUDY SUMMARY

DPP-4s as effective, less risky

This observational cohort study compared DPP-4 inhibitors and sulfonylureas when combined with metformin for the treatment of T2DM.1 Outcomes were all-cause mortality, major adverse CV events (defined as hospitalization for ischemic stroke or myocardial infarction [MI]), and hospitalizations for either heart failure or hypoglycemia. The study included data from the National Health Insurance Research Database in Taiwan on more than 70,000 patients (ages 20 and older) with diagnosed T2DM. Individuals adherent to metformin were considered to be enrolled in the cohort on the day they began using either a DPP-4 inhibitor or a sulfonylurea, in addition to metformin.

The researchers collected additional data on socioeconomic factors, urbanization, robustness of the local health care system, Charlson Comorbidity Index, adapted Diabetes Complications Severity Index, and other comorbidities and medications that could affect the outcomes of interest. Participants were then matched by propensity score into 10,089 pairs, each consisting of one DPP-4 inhibitor user and one sulfonylurea user.

After mean follow-up of 2.8 years, the investigators used Cox regression analysis to evaluate the relative hazards of the outcomes. Subgroup analysis stratified by age, sex, Charlson Comorbidity Index, hypertension, chronic kidney disease, hospitalization for heart failure, MI, and cerebrovascular disease yielded results similar to those of the primary analysis for each outcome. Similar results were also obtained when the data were analyzed without propensity-score matching.

The researchers found that users of DPP-4 inhibitors—compared with those who used sulfonylureas—had a lower risk for all-cause mortality (366 vs 488 deaths; hazard ratio [HR], 0.63; number needed to treat [NNT], 117), major cardiac events (209 vs 282 events; HR, 0.68; NNT, 191), ischemic stroke (144 vs 203 strokes; HR, 0.64; NNT, 246), and hypoglycemia (89 vs 170 events; HR, 0.43; NNT, 201). There were no significant differences in the occurrence of MIs (69 vs 88 MIs; HR, 0.75) or the number of hospitalizations for heart failure (100 vs 100 events; HR, 0.78) between the two groups.

WHAT’S NEW

Lower risks for death, CV events, and hypoglycemia

This study found that when added to metformin, DPP-4 inhibitors were associated with lower risks for all-cause mortality, CV events, and hypoglycemia when compared to sulfonylureas. Additionally, DPP-4 inhibitors did not increase the risk for heart failure hospitalization. A recent multicenter observational study of nearly 1.5 million patients on the effects of incretin-based treatments (including DPP-4 inhibitors and GLP-1 agonists) found no increased risk for heart failure hospitalization with DPP-4 inhibitors, compared to other combinations of oral T2DM agents.8

 

 

 

CAVEATS

Did unmeasured confounders play a role?

Unmeasured confounders potentially bias all observational population cohort results. In this particular study, there may have been unmeasured but significant patient factors that providers used to choose diabetes medications. Also, the study did not evaluate diabetes control, although previous studies have shown similar glucose control between sulfonylureas and DPP-4 inhibitors when added to metformin.6

Another caveat is that the results from this study group may not be generalizable to other populations due to physiologic differences. People of Asian ancestry are at risk for T2DM at a lower BMI than people of European ancestry, which could affect the outcomes of interest.9

Furthermore, the study did not evaluate outcomes based on whether patients were taking first-, second-, or third-generation sulfonylureas. Some sulfonylureas (eg, glyburide) carry a higher risk for hypoglycemia, which could bias the results.10

Lastly, the study only provides guidance when choosing between a sulfonylurea and a DPP-4 inhibitor for secondline pharmacotherapy. The GRADE trial, due to be completed in 2023, is comparing sulfonylureas, DPP-4 inhibitors, GLP-1 agonists, and insulin as add-on medications to metformin; it may provide more data on which to base treatment decisions.11

CHALLENGES TO IMPLEMENTATION

DPP-4s are more expensive

Sulfonylureas and DPP-4 inhibitors are both available as generic medications, but the cost of DPP-4 inhibitors remains significantly higher.12 Higher copays and deductibles could affect patient preference. For patients without health insurance, sulfonylureas are available on the discounted drug lists of many major retailers, while DPP-4 inhibitors are not.

ACKNOWLEDGEMENT

The PURLs Surveillance System was supported in part by Grant Number UL1RR024999 from the National Center For Research Resources, a Clinical Translational Science Award to the University of Chicago. The content is solely the responsibility of the authors and does not necessarily represent the official views of the National Center For Research Resources or the National Institutes of Health.

Copyright © 2017. The Family Physicians Inquiries Network. All rights reserved.

Reprinted with permission from the Family Physicians Inquiries Network and The Journal of Family Practice. 2017;66(1):42-44.

 

A 58-year-old woman with T2DM and heart failure returns to your office for follow-up. She has been on the maximum dose of metformin alone for the past six months, but her A1C is now 7.8%. She wants to avoid injections. What do you recommend?

There is surprisingly little consensus about what to add to metformin for patients with T2DM who require a second agent to achieve their glycemic goal. Attaining glycemic control earlier in the course of the disease may lead to reduced overall cardiovascular (CV) risk, so the choice of a second drug is an important one.2 While the proven mortality benefit, wide availability, and low cost of metformin make it well-established as initial pharmacotherapy, no second-choice drug has amassed enough evidence of benefit to become the add-on therapy of choice.

The professional societies are of little assistance; dual-therapy recommendations from the American Diabetes Association and the European Association for the Study of Diabetes do not specify a preference.3 Although the American Association of Clinical Endocrinologists/American College of Endocrinology suggest a hierarchy of choices, it is based on expert consensus recommendations.4

A look at the options

Options for add-on therapy include sulfonylureas, thiazolidines, DPP-4 inhibitors, sodium glucose cotransporter 2 inhibitors, glucagon-like peptide 1 (GLP-1) agonists, and insulin. Providers frequently prescribe sulfonylureas after metformin because they are low in cost, have long-term safety data, and are effective at lowering A1C. They work by directly stimulating insulin secretion via pancreatic ß-cells in a glucose-independent manner. But as a 2010 meta-analysis revealed, sulfonylureas carry significant risk for hypoglycemia (relative risk [RR], 4.57) and weight gain (average, 2.06 kg), compared to placebo.5

DPP-4 inhibitors, on the other hand, induce insulin secretion in a glucose-dependent manner through an incretin mechanism. Combined with metformin, they provide glucose control similar to that achieved with the combination of a sulfonylurea and metformin.6 DPP-4 inhibitors were initially found to be associated with fewer CV events and less hypoglycemia than sulfonylureas but were subsequently linked to an increased risk for heart failure–related hospitalization.7

A recent study provides more data on the effects of DPP-4s added to metformin.1

STUDY SUMMARY

DPP-4s as effective, less risky

This observational cohort study compared DPP-4 inhibitors and sulfonylureas when combined with metformin for the treatment of T2DM.1 Outcomes were all-cause mortality, major adverse CV events (defined as hospitalization for ischemic stroke or myocardial infarction [MI]), and hospitalizations for either heart failure or hypoglycemia. The study included data from the National Health Insurance Research Database in Taiwan on more than 70,000 patients (ages 20 and older) with diagnosed T2DM. Individuals adherent to metformin were considered to be enrolled in the cohort on the day they began using either a DPP-4 inhibitor or a sulfonylurea, in addition to metformin.

The researchers collected additional data on socioeconomic factors, urbanization, robustness of the local health care system, Charlson Comorbidity Index, adapted Diabetes Complications Severity Index, and other comorbidities and medications that could affect the outcomes of interest. Participants were then matched by propensity score into 10,089 pairs, each consisting of one DPP-4 inhibitor user and one sulfonylurea user.

After mean follow-up of 2.8 years, the investigators used Cox regression analysis to evaluate the relative hazards of the outcomes. Subgroup analysis stratified by age, sex, Charlson Comorbidity Index, hypertension, chronic kidney disease, hospitalization for heart failure, MI, and cerebrovascular disease yielded results similar to those of the primary analysis for each outcome. Similar results were also obtained when the data were analyzed without propensity-score matching.

The researchers found that users of DPP-4 inhibitors—compared with those who used sulfonylureas—had a lower risk for all-cause mortality (366 vs 488 deaths; hazard ratio [HR], 0.63; number needed to treat [NNT], 117), major cardiac events (209 vs 282 events; HR, 0.68; NNT, 191), ischemic stroke (144 vs 203 strokes; HR, 0.64; NNT, 246), and hypoglycemia (89 vs 170 events; HR, 0.43; NNT, 201). There were no significant differences in the occurrence of MIs (69 vs 88 MIs; HR, 0.75) or the number of hospitalizations for heart failure (100 vs 100 events; HR, 0.78) between the two groups.

WHAT’S NEW

Lower risks for death, CV events, and hypoglycemia

This study found that when added to metformin, DPP-4 inhibitors were associated with lower risks for all-cause mortality, CV events, and hypoglycemia when compared to sulfonylureas. Additionally, DPP-4 inhibitors did not increase the risk for heart failure hospitalization. A recent multicenter observational study of nearly 1.5 million patients on the effects of incretin-based treatments (including DPP-4 inhibitors and GLP-1 agonists) found no increased risk for heart failure hospitalization with DPP-4 inhibitors, compared to other combinations of oral T2DM agents.8

 

 

 

CAVEATS

Did unmeasured confounders play a role?

Unmeasured confounders potentially bias all observational population cohort results. In this particular study, there may have been unmeasured but significant patient factors that providers used to choose diabetes medications. Also, the study did not evaluate diabetes control, although previous studies have shown similar glucose control between sulfonylureas and DPP-4 inhibitors when added to metformin.6

Another caveat is that the results from this study group may not be generalizable to other populations due to physiologic differences. People of Asian ancestry are at risk for T2DM at a lower BMI than people of European ancestry, which could affect the outcomes of interest.9

Furthermore, the study did not evaluate outcomes based on whether patients were taking first-, second-, or third-generation sulfonylureas. Some sulfonylureas (eg, glyburide) carry a higher risk for hypoglycemia, which could bias the results.10

Lastly, the study only provides guidance when choosing between a sulfonylurea and a DPP-4 inhibitor for secondline pharmacotherapy. The GRADE trial, due to be completed in 2023, is comparing sulfonylureas, DPP-4 inhibitors, GLP-1 agonists, and insulin as add-on medications to metformin; it may provide more data on which to base treatment decisions.11

CHALLENGES TO IMPLEMENTATION

DPP-4s are more expensive

Sulfonylureas and DPP-4 inhibitors are both available as generic medications, but the cost of DPP-4 inhibitors remains significantly higher.12 Higher copays and deductibles could affect patient preference. For patients without health insurance, sulfonylureas are available on the discounted drug lists of many major retailers, while DPP-4 inhibitors are not.

ACKNOWLEDGEMENT

The PURLs Surveillance System was supported in part by Grant Number UL1RR024999 from the National Center For Research Resources, a Clinical Translational Science Award to the University of Chicago. The content is solely the responsibility of the authors and does not necessarily represent the official views of the National Center For Research Resources or the National Institutes of Health.

Copyright © 2017. The Family Physicians Inquiries Network. All rights reserved.

Reprinted with permission from the Family Physicians Inquiries Network and The Journal of Family Practice. 2017;66(1):42-44.

References

1. Ou SM, Shih CJ, Chao PW, et al. Effects of clinical outcomes of adding dipeptidyl peptidase-4 inhibitors versus sulfonylureas to metformin therapy in patients with type 2 diabetes mellitus. Ann Intern Med. 2015;163:663-672.
2. Hayward RA, Reaven PD, Wiitala WL, et al. Follow-up of glycemic control and cardiovascular outcomes in type 2 diabetes. N Engl J Med. 2015;372:2197-2206.
3. American Diabetes Association. Standards of Medical Care in Diabetes—2016. Diabetes Care. 2016;39(suppl 1).
4. Garber AJ, Abrahamson MJ, Barzilay JI, et al. Consensus Statement by the American Association of Clinical Endocrinologists and American College of Endocrinology on the Comprehensive Type 2 Diabetes Management Algorithm—2016 Executive Summary. Endocr Pract. 2016;22: 84-113.
5. Phung OJ, Scholle JM, Talwar M, et al. Effect of noninsulin antidiabetic drugs added to metformin therapy on glycemic control, weight gain, and hypoglycemia in type 2 diabetes. JAMA. 2010;303:1410-1418.
6. Gallwitz B, Rosenstock J, Rauch T, et al. 2-year efficacy and safety of linagliptin compared with glimepiride in patients with type 2 diabetes inadequately controlled on metformin: a randomised, double-blind, non-inferiority trial. Lancet. 2012;380:475-483.
7. Scirica BM, Bhatt DL, Braunwald E, et al. Saxagliptin and cardiovascular outcomes in patients with type 2 diabetes mellitus. N Engl J Med. 2013;369:1317-1326.
8. Filion KB, Azoulay L, Platt RW, et al. A multicenter observational study of incretin-based drugs and heart failure. N Engl J Med. 2016;374:1145-1154.
9. Chan JC, Malik V, Jia W, et al. Diabetes in Asia: epidemiology, risk factors, pathophysiology. JAMA. 2009;301:2129-2140.
10. Gangji AS, Cukierman T, Gerstein HC, et al. A systematic review and meta-analysis of hypoglycemia and cardiovascular events: a comparison of glyburide with other secretagogues and with insulin. Diabetes Care. 2007;30:389-394.
11. Nathan DM, Buse JB, Kahn SE, et al. Rationale and design of the glycemia reduction approaches in diabetes: a comparative effectiveness study (GRADE). Diabetes Care. 2013;36:2254-2261.
12. GoodRx. Gliptins. www.goodrx.com/gliptins. Accessed January 4, 2017.

References

1. Ou SM, Shih CJ, Chao PW, et al. Effects of clinical outcomes of adding dipeptidyl peptidase-4 inhibitors versus sulfonylureas to metformin therapy in patients with type 2 diabetes mellitus. Ann Intern Med. 2015;163:663-672.
2. Hayward RA, Reaven PD, Wiitala WL, et al. Follow-up of glycemic control and cardiovascular outcomes in type 2 diabetes. N Engl J Med. 2015;372:2197-2206.
3. American Diabetes Association. Standards of Medical Care in Diabetes—2016. Diabetes Care. 2016;39(suppl 1).
4. Garber AJ, Abrahamson MJ, Barzilay JI, et al. Consensus Statement by the American Association of Clinical Endocrinologists and American College of Endocrinology on the Comprehensive Type 2 Diabetes Management Algorithm—2016 Executive Summary. Endocr Pract. 2016;22: 84-113.
5. Phung OJ, Scholle JM, Talwar M, et al. Effect of noninsulin antidiabetic drugs added to metformin therapy on glycemic control, weight gain, and hypoglycemia in type 2 diabetes. JAMA. 2010;303:1410-1418.
6. Gallwitz B, Rosenstock J, Rauch T, et al. 2-year efficacy and safety of linagliptin compared with glimepiride in patients with type 2 diabetes inadequately controlled on metformin: a randomised, double-blind, non-inferiority trial. Lancet. 2012;380:475-483.
7. Scirica BM, Bhatt DL, Braunwald E, et al. Saxagliptin and cardiovascular outcomes in patients with type 2 diabetes mellitus. N Engl J Med. 2013;369:1317-1326.
8. Filion KB, Azoulay L, Platt RW, et al. A multicenter observational study of incretin-based drugs and heart failure. N Engl J Med. 2016;374:1145-1154.
9. Chan JC, Malik V, Jia W, et al. Diabetes in Asia: epidemiology, risk factors, pathophysiology. JAMA. 2009;301:2129-2140.
10. Gangji AS, Cukierman T, Gerstein HC, et al. A systematic review and meta-analysis of hypoglycemia and cardiovascular events: a comparison of glyburide with other secretagogues and with insulin. Diabetes Care. 2007;30:389-394.
11. Nathan DM, Buse JB, Kahn SE, et al. Rationale and design of the glycemia reduction approaches in diabetes: a comparative effectiveness study (GRADE). Diabetes Care. 2013;36:2254-2261.
12. GoodRx. Gliptins. www.goodrx.com/gliptins. Accessed January 4, 2017.

Issue
Clinician Reviews - 27(2)
Issue
Clinician Reviews - 27(2)
Page Number
20-22
Page Number
20-22
Publications
Publications
Topics
Article Type
Display Headline
Need an Add-on to Metformin? Consider This
Display Headline
Need an Add-on to Metformin? Consider This
Sections
Disallow All Ads
Content Gating
No Gating (article Unlocked/Free)
Alternative CME
Article PDF Media